0% found this document useful (0 votes)
39 views

Questions and Answers

MCQ questions with answers

Uploaded by

eliza642531
Copyright
© © All Rights Reserved
We take content rights seriously. If you suspect this is your content, claim it here.
Available Formats
Download as DOCX, PDF, TXT or read online on Scribd
0% found this document useful (0 votes)
39 views

Questions and Answers

MCQ questions with answers

Uploaded by

eliza642531
Copyright
© © All Rights Reserved
We take content rights seriously. If you suspect this is your content, claim it here.
Available Formats
Download as DOCX, PDF, TXT or read online on Scribd
You are on page 1/ 60

Exam MCQs Faculty therapy 4 year

1. A 53-year-old man, who works as a chef, presents to accident and emergency with
sudden onset severe pain, tenderness and swelling of the first metatarsophalangeal
joint. The pain is making it difficult for him to mobilize. He has had two previous
similar episodes. Blood tests reveal a raised serum urate level. The most likely
diagnosis is:
A. Gout
B. Pseudo-gout
C. Septic arthritis
D. Reactive arthritis
E. Osteoarthritis

2. Which of the following features is characteristic of osteoarthritis?


A. Pannus formation
B. Heberdon nodes
C. Baker cyst
D. Sacroiliitis
E. Tophi

3. A 36-year-old woman presents to the rheumatology outpatient clinic with a two-month history of stiff hands and
wrists. She mentions that the pain is particularly bad for the first few hours after waking up and is affecting her work
as a dentist. On examination, the wrists, metacarpophalangeal joints and proximal interphalangeal joints are swollen
and warm. What is the most likely diagnosis?
A. Rheumatoid arthritis
B. Osteoarthritis
C. Septic arthritis
D. Polymyalgia rheumatica
E. Reactive arthritis

4. A 45-year-old woman presents to the rheumatology outpatient clinic with a three month
history of stiff hands and wrists. She mentions that the pain is particularly bad first thing in the morning. On
examination, the wrists, metacarpophalangeal joints and proximal interphalyngeal joints are swollen and warm. A
diagnosis of rheumatoid arthritis is suspected. Which of the following investigations is most specific for confirming
the diagnosis?
A. X-rays
B. Rheumatoid factor levels
C. Anti-citrullinated peptide antibody (anti-CCP) levels
D. C-reactive protein
E. Erythrocyte sedimentation rate

5. A 40-year-old woman presents to the rheumatology outpatient clinic with a three month
history of stiff hands and wrists. She mentions that the pain is particularly
bad first thing in the morning. On examination, the wrists, metacarpophalangeal
joints and proximal interphalangeal joints are swollen and warm. A diagnosis of
rheumatoid arthritis is suspected. Blood tests for rheumatoid factor return as
positive. What is the most appropriate management?
A. Non-steroidal anti-inflammatory drugs (NSAIDs)
B. Intramuscular depot injection of methylprednisolone plus NSAIDs
C. Anti-TNF therapy
D. Intramuscular depot injection of methylprednisolone plus NSAIDs and
methotrexate and sulfasalazine
E. Physiotherapy

6. A 75-year-old woman presents to accident and emergency complaining of pain


in her knees. She mentions that this has been troubling her for several months. Pain
is generally worse in the evenings and after walking. On examination, there are
palpable bony swellings on the distal interphalangeal joints of the fingers on both
hands. In addition, there is reduced range of movement and crepitus in the knees.
What is the most likely diagnosis?
A. Rheumatoid arthritis
B. Osteoarthritis
C. Reactive arthritis

1
D. Polymyalgia rheumatica
E. Gout
7. A 79-year-old woman presents to her GP with pain in the left knee. This is
particularly bad in the evenings and is stopping her from sleeping. The GP explains
that her discomfort is most likely due to osteoarthritis and arranges for her to have
an x-ray of the knee. Which of the following descriptions are most likely to describe
the x-ray?
A. Reduced joint space, subchondral sclerosis, bone cysts and osteophytes
B. Increased joint space, subchondral sclerosis, bone cysts and osteophytes
C. Reduced joint space, soft tissue swelling and peri-articular osteopenia
D. Increased joint space, soft tissue swelling and peri-articular osteopenia
E. Normal x-ray

8. A 30-year-old man presents to his GP with a 1-week history of painful, swollen knees and a painful right heel.
Further history reveals that he has been experiencingburning pains while urinating for the past 2 weeks and that
his eyes have becomered and itchy. What is the most likely diagnosis?
A. Reactive arthritis
B. Gout
C. Ankylosing spondylitis
D. Enteropathic arthritis
E. Septic arthritis

9. A 30-year-old Afro-Carribean woman presents to accident and emergency with a


1-week history of progressive shortness of breath and fever. On further
questioning, she mentions that her hands have been painful and stiff over the past
few months and she has been having recurrent mouth ulcers. Chest x-ray confirms
bilateral
pleural effusions and blood tests reveal a raised ESR and Anti-dsDNA (double-stranded DNA) in abnormal titre.
What is the most likely diagnosis?
A. Systemic lupus erythematosus
B. Systemic sclerosis
C. Sjцgren’s syndrome
D. Discoid lupus
E. Beзhet’s disease

10. A 34-year-old Afro-Carribean woman has been admitted for management and investigation of increasing
shortness of breath. On further questioning, shementions that her hands have been painful and stiff over the past
few months and she has been having recurrent mouth ulcers. Chest x-ray confirms bilateral pleural effusions and
blood tests reveal a raised ESR and a normal CRP. A diagnosis of systemic lupus erythematosus (SLE) is suspected
and a full autoantibody screen is
sent to the laboratory. Which of the following auto-antibodies is most specific to the suspected diagnosis?
A. Anti-nuclear antibody
B. Rheumatoid factor
C. Anti-double stranded DNA antibody
D. Anti-centromere antibody
E. Anti-mitochondrial antibody

11. Which of the following is a distinguishing feature between drug induced and idiopathic SLE?

A) In drug-induced lupus, there is an absence of antibodies to double stranded DNA.

B) In drug-induced lupus, there are increased levels of complement.

C) In idiopathic SLE, a butterfly facial rash is seen.

D) Renal and CNS involvement are common with drug-induced SLE.

E) There are no differences seen between the two conditions.

2
12. A74-year-old woman has pain in her left hand and right knee, which started months ago, and

3
is now interfering with her activities. The paingets worse as the day progresses. There is no
history of any trauma, and she is otherwisewell. Taking over-the-counter acetaminophen
usually relieves the pain. On examination, thereis bony soft tissue swelling of her second and
third DIP joints in the left hand and crepitusover the right knee with flexion. There is noerythema or joint effusion.
Which of the followingis the most likely explanation for thejoint pain of osteoarthritis?
(A) synovial inflammation is not the cause
(B) ligament inflammation is a common cause
(C) clinically visible (via plain x-ray) fractures are a common cause of pain
(D) osteophytes can cause pain
(E) muscles are not involved

13. A woman presents with a facial rash,arthralgias, and fatigue. The rash on her face is
erythematous and raised, her heart and lungsare normal, and wrists are swollen and
tender on palpation. She has mild thrombocytopenia(90,000/mL). Which of the following
is the most appropriate initial autoantibody test?
(A) anti-double-stranded (ds) deoxyribonucleic acid (DNA)
(B) anti-Sm
(C) anti-Ro or La
(D) ANA
(E) antiphospholipid antibodies (lupus anticoagulant)

14. A 32 year old woman complains of polyarticular symmetric arthritis and a diffuse maculopapular rash that flares
up with exposure to sun light. She is afebrile. Physical examination reveals tenosynovitis of her wrists. Which one
of the following is the most likely diagnosis?

a) Gonococcal arthritis

b) Reiter's syndrome

c) Rheumatoid arthritis

d) Systemic lupus erythematosus

15. A 35 year old white female comes to your office with a 3-month history of the gradual onset of pain and
tenderness in the wrists and hands. She also complains of 1 hour of morning stiffness. She denies rash, fever, or skin
changes. On physical examination she has symmetric swelling of the proximal interphalangeal joints and
metacarpophalangeal joints. Motion of these joints is painful. Her CBC is normal, but the erythrocyte sedimentation
rate is elevated at 40 mm/hr. Test positive for anti-CCP The most likely diagnosis in this patient is:

a) Rheumatoid arthritis

b) Systemic lupus erythematosus

c) Sarcoidosis

d) Lyme disease

e) Calcium pyrophosphate desposition disease

16. A 40 year old woman presents with painful swelling of the joints of her fingers, excessive fatigue and a
malar rash.

Which one of the following diseases is most likely?

a) Psoriasis

b) Pseudogout

c) Systemic lupus erythematosus (SLE)

d) Rheumatoid arthritis

e) Serum sickness

4
17. A 54 year old man presents with acute onset of painful first metatarsophalangeal joint. It is swollen, red and
tender. This is his first episode. You decide to order a serum uric acid and a 24 urine for uric acid.

You then prescribe for him which of the following medicines?

a) Colchicine

b) Indomethacin

c) Allopurinol

d) Probenacid

18. A 62 year old white male comes to your office with pain and swelling of the left great toe at the
metatarsophalangeal joint. Examination shows it is erythematous, warm, swollen, and tender to touch. The patient
has a history of diabetes mellitus controlled by diet, and hypertension. His medications include hydrochlorothiazide,
25 mg/day. A CBC and blood chemistry profile are normal, except for a uric acid level of 9.2 mg/dL (N 3.6-8.5).
Which one of the following is true in this situation?

a) This attack should resolve spontaneously in 3-4 days

b) Allopurinol (Zyloprim) therapy should be started

c) The elevated uric acid level establishes the diagnosis of gout

d) Intra-articular steroid injection should be avoided

e) Stopping the hydrochlorothiazide may control the hyperuricemia

19. A 53 year old presents to your office with pain and stiffness in both hands and knees of 6 months duration.
You suspect rheumatoid arthritis. Which of the following findings on your history and physical examination is
most distinctive with a diagnosis?

a) Unilateral joint tenderness/effusions

b) Maculopapular rash

c) Joint stiffness worse in the morning

d) Hepatosplenomegaly

20. Which is the most common site of subcutaneous nodules in rheumatoid arthritis –
a) Elbow b) Wrist
c) Achilles tendon d) Occiput

21. A 68 year old patient presents with an acutely red and swollen right great toe without history of trauma. Which
one of the following findings is most useful for making a diagnosis in this patient?

a) Joint fluid aspiration

b) Good response to colchicine trial

c) Radiograph showing marginal joint erosion in the first metatarsophalangeal (MTP) joint

d) An associated right ankle effusion

e) Painless elbow nodule

22. A 55-year-old man presents with complaints of an acute gouty arthritis attack. His left big toe is red, swollen,
andtender to the touch. This is his first attack. He has no history of peptic ulcer disease. His uric acid level is 8
mgldl. Which of the following agents would you recommend for treatment?

5
A. Prednisone
B. Colchicine
C. Allopurinol
D. Indomethacin
E. Probenecid

23. A 60-year-old man presents with pain, swelling, and redness of the first

metatarsal phalangeal joint. His cardiologist recently prescribed

hydrochlorothiazide for his hypertension. The most likely diagnosis is

A) Gout

B) Degenerative joint disease

C) Rheumatoid arthritis

D) Osteomyelitis

E) Morton’s neuroma

24. Type of anemia seen in Rheumatoid arthritis is -


a) Microcytic hypochromic anaemia
b) Macrocytic hypochromic anaemia
c) Normocytic hypochromic anaemia
d) Normocytic normochromic anaemia

25. A74-year-old woman has pain in her left hand and right knee, which started months ago, and
is now interfering with her activities. The paingets worse as the day progresses. There is no
history of any trauma, and she is otherwise well. Taking over-the-counter acetaminophen
usually relieves the pain. On examination, thereis bony soft tissue swelling of her second and
third DIP joints in the left hand and crepitus over the right knee with flexion. There is no erythema or joint effusion.
Which of the followingis the most likely explanation for thejoint pain of osteoarthritis?
(A) synovial inflammation is not the cause
(B) ligament inflammation is a common
cause
(C) clinically visible (via plain x-ray)
fractures are a common cause of pain
(D) osteophytes can cause pain
(E) muscles are not involved

26. A29-year-old woman develops painful swellingof both hands. She is also very stiff in the morning.
Physical examination reveals involvementof the proximal interphalangeal joints and
metacarpophalangeal (MCP) joints. Her RF is positive and ANA is negative. Which of thefollowing medications is
most likely toimprove her joint pain symptoms?
(A) D-penicillamine
(B) an antimalarial
(C) methotrexate
(D) NSAID or aspirin
(E) gold

27. A72-year-old man injures his right knee in a caraccident, and now it is swollen and extremely
painful to bend the knee. X-rays of the knee ruleout a fracture, and joint fluid aspiration reveals

6
an opaque-colored fluid containing rhomboidcrystals with weak-positive birefringence. Which
of the following is the most appropriate nextstep in management?
(A) oral prednisone
(B) intravenous antibiotics
(C) oral NSAIDs
(D) acetaminophen
(E) allopurinol

28. A 25-year-old woman develops painful swelling of both hands and wrists. She is also
very stiff in the morning. Physical examination reveals erythema, swelling and joint line tenderness
of the proximal interphalangeal, MCP,and wrists joints. Her RF is positive, ANA is negative, and x-rays of the
hands show early erosive joint changes. Which of the following medications is most likely to prevent progressionof
disease?
(A) D-penicillamine
(B) antimalarial
(C) methotrexate
(D) NSAID or aspirin
(E) gold

29. Which part of the spine is most commonly affected in Rheumatoid arthritis -
a) Cervical b) Lumbar
c) Thoracic d) Sacral

30. Which of the following is true regarding Rheumatoid arthritis -


a) Typically involves small and large joints symmetrically but spares the cervical spine
b) Causes pleural effusion with low sugar
c) Pulmonary nodules are absent
d) Enthesopathy prominent

31. Which of the following is the most specific test for Rheumatoid arthritis -
a) Anti-ccp antibody
b) Anti IgA antibody
c) Anti IgM antibody
d) Anti IgG antibody

32. Type of anemia seen in Rheumatoid arthritis is -


e) Microcytic hypochromic anaemia
f) Macrocytic hypochromic anaemia
g) Normocytic hypochromic anaemia
h) Normocytic normochromic anaemia

33. The most common cardiac involvement in


rheumatoid arthritis - a) Pancarditis b) Pericarditis
c) Myocarditis d) Endocarditis

34. Drugs used in treatment of acute gout –


a) Allopurinol b) Colchicinec) Pamidronate d) Methotrexate
35. Which part of the spine is most commonly affected in Rheumatoid arthritis - a) Cervical b) Lumbar
c) Thoracic d) Sacral
36. Part of spine, involved in Rheumatoid arthritis is - a) Cervical b) Thoracic c) Lumbar d) Sacral

37. Drugs used in treatment of acute gout –


a) Allopurinol

7
b) Colchicine

8
c) Pamidronate
d) Methotrexate
38. Proximal interphalangeal, distal interphalangeal & 1st metacarpophalangeal joint involvement and sparing
of wrist is a feature of –
a) Rheumatoid arthritis
b) Pseudogout
c) Psoriatic arthropathy
d) Osteoarthritis
39. Gout is a disorder of-
a) Purine metabolism
b) Pyrimidine metabolism
c) Ketone metabolism
d) Protein metabolism
40. Indications for use of cyclophosphamide in SLE
a) Arthritis
b) Anti DNA antibody + ive patients
c) Diffuse prolifereative glomerulonephritis
d) Oral ulcers
e) Discoid lipus

41. A 67-year-old male treated for hypertension and hyperlipidemia complains of nagging right knee pain that is
worse in the evening. The pain has been present for several months and it seems to limit his physical activities. His
blood pressure is 160/100 mmHg and his heart rate is 70/min. His BMI is 32 kg/m2, and palpation of the knee
reveals a cool joint with bony tenderness. His blood cholesterol level is 200 mg/dl and his serum uric acid level is
9.0 mg/dl. Which of the following additional findings is likely on further examination of the right knee?

A. Soft tissue swelling


B. Painful tibial tuberosity
C. Palpable popliteal mass
D. Bony crepitus
E. Subcutaneous nodules

42. Gout is a disorder of-


a) Purine metabolism b) Pyrimidine metabolism c) Ketone metabolism d) Protein metabolism

43. All of the following statements about primary Gouty arthritis are true, Except -
a) 90% of cases are caused by over production of uric acid
b) Uric acid levels may be normal at the time of an acute attack
c) Men are more commonly affected than women (Male > Females)
d) Definitive diagnosis requires aspiration of synovial fluid
44. False regarding gouty arthritis is -
a) Synovial fluid analysis is diagnostic
b) Allopurinol is the treatment of choice in acute gout
c) Arthritis occurs after long attack of hyperuricemia
d) Level of uric acid in blood and severity of gout has good correlation
45. Most specific diagnostic finding in gout is -
a) Uric acid crystals in urine
b) Raised serum uric acid
c) Presence of calcium pyrophosphate crystals in synovial fluid
d) Presence of monosdium crystals in synovial fluid

46. Most common manifestation of cardiac lupus is -


a) Myocarditis b) Libman-Sacks endocardits c) Pericarditis d) Aortic regurgitation

40. A 23 years old woman has experienced episodes < myalgias, pleural effusion, pericarditis an arthralgias without
joint deformity over course < several years. The best laboratory screening test i diagnose her disease would be -

9
a) CD lymphocyte count
b) Erythrocyte sedimentation rate
c) Antinuclear antibody
d) Assay for thyroid hormones

47. Anti ds DNA antibody is specific for -


a) SLE
b) Systemic sclerosi
c) CREST syndrome
d) Sjogren’s syndroi
e) Wegener’s granulomatosis

48. Indications for use of cyclophosphamide in SLE


a) Arthritis
b) Anti DNA antibody + ive patients
c) Diffuse prolifereative glomerulonephritis
d) Oral ulcers
e) Discoid lipus

49. Anti-double stranded DNA is highly specific for -


a) Systemic sclerosis
b) S.L.E.
c) Polymyositis
d) Rheumatic sclerosis
50.Specific antibody for SLE is -
a) Anti-Ro b)Anti-Jo
c) Anti-Sm d) Anti ds DNA
e) Anti-La
51.A 23-year old woman has experienced episodes of myalgias, pleural effusion, pericarditis and arthralgias without
joint deformity over course of several years. The best laboratory screening test to diagnose her disease would be -
a) CD4 lymphocyte count
b) Erythrocyte sedimentation rate
c) Antinuclear antibody
d) Assay for thyroid hormones
52.Characteristic feature of SLE is -
a) Uveitis b) Joint deformity
c) Polyserositis d) Cavitating lesion is lung
53.Autoimmune destruction of platelet is seen in-
a) SLE b) Rheumatoid arthritis
c) Reiter disease d) Polyarteritis nodosa

54. Deposition of anti ds DNA Ab in kidney, skin, choroid plexus and joints is seen in-
a) SLE b) Good pasture
c) Scleroderma d) Raynauds disease

55. A middle aged female presents with polyarthritis with elevated rheumatoid factor ANA analevels, which
among the following will help you to differentiate rheumatoid arthritis from SLE ?
a) Soft tissue swelling at the proximal inter phalangial joint
b) Juxta articular osteoporosis or X-ray
c) Articular erosions on X-ray
d) d) Elevated ESR

10
56. Laboratory signs in rheumatoid arthritis for monitoring activity of the process:
a. increasing the level of CRP;
b.high ESR;
c. increase the level of anti-CCP antigen;
d.leukocytosis,titer SLA-O.

57. Rheumatoid arthritis often suffer from:


a. girls and young women aged 20-30 years
b. middle-aged women aged 35-55 years
c. old women and the elderly
d. young men aged 20-30 years

58. In rheumatoid arthritis, specific autoantibodies to pathogenesis are:


a.antistreptolysin O
b. antinuclear factor
c. antibodies to the circulatingcitruline peptide
d. an antibody to DNA
59. It is characteristic of rheumatoid arthritis:
a. symmetrical inflammation of more than 3 peripheral joints
b. asymmetric inflammation of 2-3 large joints
c. asymmetrical inflammation of the 1st large joints
d. symmetrical inflammation of the sacroiliac joint

60. In rheumatoid arthritis occurs:


a. symmetric polyarthritis
b. asymmetric arthritis
c. an asymmetric oligoarthritis
d. asymmetrical monoarthritis

61. For rheumatoid arthritis is characterized by predominant involvement of:


a. the large joints
b. the joints of the lower extremities
c. the knee and small joints of the hands and feet
d. joints of the spine
e. I metatarsophalangeal joints

62. In rheumatoid arthritis, the most commonly affected:


a. knee joints
b. the sacroiliac joints
c. the small joints of the hands
d. small joints of the feet
e. I metatarsophalangeal joints

63. In rheumatoid arthritis clinical signs of irreversible joint damage are:


a) chondritis
b) synovitis
c) ankylosis
d) achillitis

64. In late-stage rheumatoid arthritis are the following features of a symmetric polyarthritis:
a) exudative signs of potentially reversible

11
b) irreversible exudative signs
c) potentially reversible proliferative (fibrotic, sclerotic) signs
d) irreversible proliferative (fibrotic, sclerotic) signs and ankylosing
65. For the early stages of rheumatoid arthritis are characterized morning stiffness duration:
a) up to 30 minutes
b) from 30 minutes to 1 hour
c) over 2 hours
d) during the day
e) from 30 sec to 1 min
66. Late stages of rheumatoid arthritis are characterized morning stiffness duration:
a) up to 30 minutes
b) from 30 minutes to 1 hour
c) over 2 hours, sometimes within days
d) from 30 sec to 1 min

67. In rheumatoid arthritis symptoms such as rheumatoid nodules, muscle inflammation, lymphadenopathy,
rheumatoid vasculitis, visceritis, nervous system, eyes, and blood system, as well as fever and weight loss
are:
a) a complication of the disease
b) a low disease activity
c) the systemic manifestations of the disease
d) transformation of a systemic lupus erythematosus
68. The most frequent type of systemic manifestations of rheumatoid arthritis are:
a) rheumatoid nodules
b) muscle inflammation
c) lymphadenopathy
d) rheumatoidvasculitis

69. Kidney damage in rheumatoid arthritis is manifested most often:


a) latent glomerulonephritis
b) nephrotic glomerulonephritis
c) secondary amyloidosis of the kidneys
d) pyelonephritis
e) urolithiasis

70. The most specific laboratory parameters of rheumatoid arthritis is the detection of a high level of blood:
a) C-reactive protein
b) uric acid
c) factor and antinuclear antibodies to DNA
d) rheumatoid factors and antibodies to circulating citrulline peptide

71. The most specific radiological sign of rheumatoid arthritis is:


a) periarticular osteoporosis
b) osteophytes and osteosclerosis
c) joint space narrowing
d) marginal bone erosion

72. Treatmen of gout:


a) NSAIDs, corticosteroids;
b)B-blocers;
c) H2inhibitors, NSAIDs;
d) all of the above

12
73. Select the "basic" drug in the treatment of rheumatoid arthritis:

13
a) benzylpenicillin
b) Biseptol (septran)
c) methotrexate
d) chondroitin sulfate

74. SLE is more common in:


a) boys and young men
b) girls and young women
c) older men
d) elderly women
75. What is the natural factor can exacerbate SLE:
a) solar radiation
b) the fog
c) magnetic storms
d) high humidity
76. The specific type of skin lesions in SLE is:
a) vitiligo
b) hyperpigmentation
c) periorbital edema
d) erythematous skin rashes on the face of the type "butterfly"
77. Photosensitivity (increased skin sensitivity to sunlight) - is a particular feature:
a) rheumatoid arthritis
b) gout
c) osteoarthritis
d) systemic lupus erythematosus

78. A 40-year-old woman complains of diffuse symmetric joint pain that is worse in the morning but improves as
the day progresses. Examination shows inflammation of the proximal interphalangeal and metacarpophalangeal
(MCP) joints. The most likely diagnosis is

A) Reiter’s syndrome

B) Rheumatoid arthritis

C) Polymyalgia rheumatica

D) LE

E) Osteoarthritis

79. A 45-year-old female complaints of increasing widespread joint pains which are worse in the evening after a
stressful day at work. She de- scribes puffy hands and feet and a painful neck. Her concentration is very poor and
she has recently suffered from marital problems. Rheumatoid factor
is mildly positive. Examination reveals an increased body mass index and global restriction of movement due to
pain, but no synovitis. Which of the following investigations would be useful in this case:
a) anti-cycliccitrullinated peptide antibody;
b) ultrasound scan hands and feet;
c) anti-JO-I antibody;
d) erythrocyte sedimentation rate and CRP;
80. A 64 year old male presents with a symmetrical arthritis of both hands with early morning stiffness. His X-rays
of his hands done before clinic show the following: periarticular osteoporosis of the metacarpeal joints; erosion on
the ulnar styloid; symmetrical distribution of disease; reduced joint space at the carpal bones. What is the most
likely diagnosis:
a) ankylosing spondylitis;
b) rheumatoid arthritis;

14
c) gout;
d) secondary osteoarthritis;
75. Osteoarthritis is characterized by all signs except:
a) degradation of articular cartilage and alterations in other joint tissues;
b) joint space narrowing, subchondral sclerosis, subchondral cysts, and pe-
ripheral osteophytes;
c) pain on movement, typically occurring when movement is initiated or
when the patient begins to walk;
d) morning stiffness in and around the joints, lasting at least 1 h before
maximal improvement;
81. A 25-year-old lady with known systemic lupus erythematosus (SLE)
presents with the nephrotic syndrome. A renalbiopsy is performed and this
confirms diffuse proliferative glomeronephritis (WHO Class IV). Which of the
following treatment regimens would you advise:
Variants of answer:
a) azathioprine alone;
b) prednisolone alone;
c) azathioprine and prednisolone;
d) prednisolone and intravenous cyclophosphamide;

82. Rheumatoid arthritis is an autoimmune disease that is characterized by:


a) synovial inflammation and hyperplasia;
b) autoantibody production;
c) cartilage and bone destruction;
d) systemic features, including cardiovascular, pulmonary, psychological and skeletal disorders;
83. Which of the following is a first line drug for patient with rheumatoid arthritis:
a) ibuprofen;
b) methotrexate;
c) glucosamine sulfates;
d) pantoprazole;
84. Osteoarthritis is characterized by all signs except:
a) degradation of articular cartilage and alterations in other joint tissues;
b) joint space narrowing, subchondral sclerosis, subchondral cysts, and pe-
ripheral osteophytes;
c) pain on movement, typically occurring when movement is initiated or
when the patient begins to walk;
d) morning stiffness in and around the joints, lasting at least 1 h before
maximal improvement;
85. A 25-year-old lady with known systemic lupus erythematosus (SLE) presents with the nephrotic syndrome. A
renal biopsy is performed and this confirms diffuse proliferative glomeronephritis (WHO Class IV). Which of the
following treatment regimens would you advise:
a) azathioprine alone;
b) prednisolone alone;
c) azathioprine and prednisolone;
d) prednisolone and intravenous cyclophosphamide;

86. Normal CRP with elevated ESR seen in?


a. RA
b. SLE
c. Scleroderma
d. Polymyalgia rheumatica

87. A young girl is admitted with joint pains and butterfly rash and positive urine proteinuria. The best test for her
diagnosis is?
a. Anti ds- DNA antibody
b. Anti - centromere antibody

15
c. Antibodies to RNP
d. Antibodies to tRNA synthetase

88. Which of the following antibodies correlates with disease activity for S.L.E
a. Anti Smith antibody
b. Anti dS DNA antibody
c. Anti Histone antibody
d. Anti Rho

89. Deposition of Anti ds DNA Ab in kidney, skin, choroid plexus and joints is seen in:
a. SLE
b. Good pasture
c. Scleroderma
d. Raynauds disease

90. Investigation of choice during follow up of patient with rheumatoid arthritis:


a. X-Ray of joints
b. ESR, Rh factor
c. Blood counts
d. Rh factor, Anti CCP antibodies

91. A patient aged 40 years having arthritis of PIP andDIP along with carpo-metacarpal joint of thumb andsparing of
wrist and metacarpo-phalangeal joint, the most likely cause is:
a. Psoriatic arthritis
b. Osteoarthritis
c. Rheumatoid arthritis
d. Pseudogout

92. Type of anemia seen in Rheumatoid arthritis:


a. Normocytic, normochromic
b. Hyperchromic, Normocytic
c. Hypochromic, normocytic
d. Hypochromic, leucopenia

93. A middle aged female presents with polyarthritis, elevated Rheumatoid factor and ANA levels . Which of the
following features will help in differentiating
Rheumatoid arthritis from SLE
a. Soft tissue swelling in PIP Joint
b. Juxta-articular osteoporosis on X ray
c. Articular erosions on X Ray
d . Elevated ESR

94. Which part of the spine is most commonly affected in Rheumatoid arthritis:
a. Cervical
b. Lumbar
c. Thoracic
d. Sacral

95. Heberden nodes are seen in:


a. Rheumatoid arthritis
b. Rheumatic arthritis
c. Osteoarthritis
d. SLE

96. In rheumatoid arthritis pathology starts in:


a. Articular cartilage
b. Capsule
c. Synovium
d. Muscle

16
97. A Carey-Coomb's murmur heard in a child with multiple joint pains is suggestive of:
a. Infective endocarditis
b. Rheumatoid arthritis
c. Rheumatic fever
d. Libman-Sacs endocarditis

98. Which of the following clinical manifestations is NOT a major criterion for diagnosing Acute Rheumatic Fever?
a) Carditis
b) Chorea
c) Elevated C-reactive protein
d) Subcutaneous nodules

99. Acute Rheumatic Fever (ARF) is primarily caused by which bacterial species?

a) Staphylococcus aureus
b) Streptococcus pneumoniae
c) Streptococcus agalactiae
d) Streptococcus pyogenes (Group A Streptococcus)

100.Molecular mimicry in Acute Rheumatic Fever refers to:

a) The ability of bacteria to mimic host cells.


b) The interaction between antibodies and antigens.
c) The cross-reactivity between bacterial antigens and host tissues.
d) The production of toxins by Streptococcus pyogenes.

101.A 20-year-old male presents with pain in the knee joints for the last two weeks. He also has mild backache
and stiffness. He had fever and diarrhoea four weeks previous to developing these complaints. The patient
is afebrile and has no diarrhoea. His laboratory investigations reveal normal ESR and elevated CRP. What
is the most probable diagnosis?
a) Rheumatoid arthritis
b) Enteropathic arthritis
c) Reactive arthritis
d) Ankylosing spondylitis

102.A young girl is admitted with joint pains and butterfly rash and positive urine proteinuria. The best test for her
diagnosis is?
a. Anti ds- DNA antibody
b. Anti - centromere antibody
c. Antibodies to RNP
d. Antibodies to tRNA synthetase

103. What is the primary treatment for Acute Rheumatic Fever to prevent complications and reduce the risk of
spreading the infection?
a) Antiviral medications
b) Antibiotics, such as penicillin
c) Corticosteroids
d) Nonsteroidal anti-inflammatory drugs (NSAIDs)

104. Which clinical manifestation of Acute Rheumatic Fever involves rapid, jerky movements and
behavioral changes?
a) Carditis
b) Arthritis
c) Chorea
d) Subcutaneous nodules
105. Which genetic marker can influence an individual's susceptibility to developing Acute Rheumatic Fever
after a streptococcal infection?

a) HLA-B27
b) Rh factor

17
c) ABO blood type
d) Genetic markers associated with the immune response
106. Which of the following is a major clinical manifestation of Acute Rheumatic Fever that involves
inflammation of the heart?
a) Chorea
b) Arthritis
c) Carditis
d) Subcutaneous nodules

107. Which bacterial virulence factor plays a crucial role in molecular mimicry and the development of
Acute Rheumatic Fever?

a) Capsule
b) M protein
c) Lipopolysaccharide (LPS)
d) Teichoic acid

108. Which of the following is a minor criterion for diagnosing Acute Rheumatic Fever?

a) Carditis
b) Chorea
c) Elevated erythrocyte count
d) Subcutaneous nodules

109. The best way to prevent rheumatic fever is:


a) Avoiding contact with infected individuals
b) Getting vaccinated
c) Early diagnosis and treatment of streptococcal infections
d) Washing hands frequently

109 ВСЕГО
ГЕМАТ
110. Which of the following is associated with an spectrin deficiency in the RBC membrane?
a) Autoimmune hemolytic anemia

b) Hereditary spherocytosis

a) Thalassemia major

c) Thalassemia minor

111. A 44-year-old Asian female presents with a two-month history of shortness of breath and lethargy. She denies
any intolerance to the cold or any changes in herweight and on examination appears slightly pale. She states that she
has recently become a vegetarian. A blood film shows the presence of elliptocytes and blood results show the
following:
Haemoglobin 9.9 g/dL
Mean cell volume (MCV) 75 fL
Ferritin Low
The most likely diagnosis is:
A. Iron deficiency anaemia
B. Sideroblastic anaemia
C. Anaemia of chronic disease
D. Thalassaemia trait
E. Hereditary elliptocytosis

112. A 47-year-old teacher complains of difficulty maintaining her concentration at work teaching secondary school
children. She states that over the last four months she has become increasingly tired and easily fatigued. She has also
noticed a tingling sensation in her fingers. A blood test reveals the following:

18
Haemoglobin 10 g/dL

MCV 103 fL

The most likely diagnosis is:

A. Hypothyroidism

B. Vitamin B12 deficiency

C. Folic acid deficiency

D. Liver disease

E. Alcohol toxicity

113. A 43-year-old woman suffers from Crohn’s disease. A blood test shows the following results:

Haemoglobin 10.5 g/dL, MCV 120 fL, Platelet count 300× 109/L

The most likely diagnosis is:

A. Vitamin B12 deficiency

B. Iron deficiency

C. Hypothyroidism

D. Folic acid deficiency

E. Anaemia of chronic disease

114. A 29-year-old woman complains of a 1-week history of weakness and malaise, she has recently become a
vegetarian and eats mostly green vegetables and drinks lots of tea during the day. She is apyrexial and has a C-
reactive protein (CRP) <5. You suspect an abnormality of the patient’s iron stores. What is the most appropriate
investigation to determine iron store levels?
A. Bone marrow biopsy
B. Serum ferritin
C. Serum transferrin
D. Total iron binding capacity
E. Serum iron

115. A 52-year-old woman presents complaining of a two-month history of increasing fatigue and numbness in both
of her arms and legs. She lives at home with her husband and has found it difficult coping with the daily activities of
living. She suffers from hypothyroidism which is well controlled with thyroid replacement medication. A peripheral
blood smear shows hypersegmented neutrophils. A blood test reveals the following:
Haemoglobin 10 g/dL
Mean corpuscular volume 110 fL
Platelets 150 × 109/L
The most likely diagnosis is:
A. Thrombotic thrombocytopenic purpura
B. Iron deficiency
C. Folic acid deficiency
D. Pernicious anaemia
E. Liver disease

116. A 65-year-old man presents with a chronic history of malaise, shortness of breath and paraesthesia in his hands.
He appears tired and pale while speaking and on examination his heart rate is 115, respiratory rate 16. A Schillings
test is positive while blood tests reveal a macrocytic anaemia and a Coombs test is negative. The most likely
diagnosis is:
A. Iron deficiency anaemia
B. Haemorrhage
C. Anaemia of chronic disease
D. Pernicious anaemia
E. Autoimmune haemolytic anaemia

19
117. A 66-year-old man presents complaining of a three-month history of weakness, tingling in the limbs and a sore
tongue. The patient notes an undesired 5 kg weight loss over 2 weeks. A peripheral blood smear shows a macrocytic
anaemia, a
Schilling test shows impaired vitamin B12 absorption and a diagnosis of pernicious anaemia is made. Which of the
following antibodies is most closely associated with pernicious anaemia?
A. Anti-mitochondrial antibodies
B. Anti-intrinsic factor antibodies
C. Anti-gliadin antibodies
D. Anti-centromere antibodies
E. Anti-smooth muscle antibodies

118. For iron deficiency anemia is characteristic: a)hypochromic anemia, b)The increase in direct bilirubin,
c)normochromic anemia, d)The increase in serum iron

119. Normal levels of Serum Iron : a)2-10 mmol / l, b)10-27mmol / l, c)15-45 mmol / l, d)26-50 mmol / l, e)8-16
mmol / l

120. Female, 55 years old, complaints of prolonged cyclical uterine bleeding during the year, weakness, dizziness.
Objective: pale skin. In the blood: Hb - 70 g/l, er.-3,2x10 12/l, CI - 0,6, leuk.-6,0x109 /l, reticulocytes - 1%;
hypochromia of erythrocytes. What is anemia in patients?

A. Chronic posthemorrhagic iron deficiency anaemia

B. Hemolytic

C. Iron-deficiency

D. B12-deficiency

121. In what situation is iron deficiency anemia apply blood transfusion:


1. women with profuse menstruation
2. Pregnant with a Hb content of more than 80 g / L
3. donors who developed anemia
4. with not abundant hemorrhoidal bleeding
5. with preanemic coma

122. Most sensitive and specific test for diagnosis of iron deficiency is -

a) Serum iron levels

b) Serum ferritin levels

c) Serum transferrin receptor population

d) Transferrin saturation

123. The criteria for recovery from iron deficiency anemia: а)appearance of reticulocytic crisis in the first 7-10 days,
b) normalization of ESR, c)The bone marrow hypoplasia, d) The total protein level

124. Which of the following findings is diagnostic of iron deficiency anemia -

a) Increased TIBC, decreased serum ferritin

b) Decreased TIBC, decreased serum ferritin

20
c) Increased TIBC, increasedserumferritin

d) Decreased TIBC, increasedsermferritin

125. A 26-year-old female is 36 weeks pregnant and presents with easy tiredness and nail brittleness. Her
Haemoglobin is 6.9 g/dl with a MCV of 63 fl. Select the most appropriate treatment for the following cases with
anaemia:

A. Parenteral Iron
B. Vitamin B12 injection
C. Blood Transfusion
D. Folate tablets

126. A 80-year-old male presents with tiredness, weakness and deteriorating confusion. On examination he is
pale and confused, has ataxia on standing with loss of vibration sensation and exaggerated leg reflexes. A
full blood count reveals a pancytopaenia. Select the most appropriate treatment for the following cases
with anaemia:

A. Parenteral Iron

B. Vitamin B12 injection

C. Blood Transfusion

D. Folate tablets

E. Oral Iron

127. Iron supplementations are prescribed: 1) for 1-2 weeks; 2) for 3-6 month, 3) for 2-3 weeks; 4) for 1 month

128. A 50-year-old woman presents with lethargy and dysphagia. Oral examination showed a smooth tongue,
angular stomatitis and koilonychia. Her haemoglobin concentration is 8g/dL with an MCV of 60fL (80-96). Choose
the SINGLE most likely cause

A. hereditary spherocytosis
B. sickle cell anaemia
C. iron deficiency anaemia

D. malaria

129. A 60-year-old lady complains of tiredness, sore tongue, and numb/tingling feet. Full blood count reveals a
haemoglobin concentration of 7g/dL (11.5-16.5) and an MCV of 115 fL (80-96).

Choose the SINGLE most likely cause

A. hereditary spherocytosis
B. pernicious anaemia
C. disseminated intravascular coagulation
D. haemolytic anaemia

130. A 24 year old woman with a seizure disorder treated with phenytoin is noted to have the following findings at
15 weeks of pregnancy:

Hemoglobin (Hgb) 93 g/L (Normal 123-157 g/L)


Hematocrit (HCT) 29% (Normal 37-46%)
Mean corpuscular volume (MCV) 105 fL (Normal 80-100)

21
Which one of the following is the most likely cause for these findings?
a) Sickle cell trait
b) Iron deficiency
c) Physiologic anemia
d) Folate deficiency
e) Thalassemia

131. A 30 year old female presents to her physician's office for an increase in fatigue over the last 2 months. On
physical examination, the physician notes delayed capillary refill in the nailbeds and a pale appearance of the inner
lining of the eyelids. A peripheral smear reveals hypochromic erythrocytes. Which one of the following tests
would be most useful in confirming the probable diagnosis?

a) Serum ferritin and total iron-binding capacity (TIBC)


b) Schilling's test
d) Hemoglobin electrophoresis
e) Serum folate and cobalamin (vitamin B12)

132. A 50-year-old woman with a long history of alcohol abuse is prescribed Phenytoin for epilepsy. Her full
blood count reveals:

Haemoglobin10.0 g/dL, MCV-122 fL, White cell count-2.2 x109/L, Platelet count-85 x109/L

What is the most likely explanation for these results?

A. Alcoholic liver disease


B. Aplastic anaemia
C. Folic acid deficiency
D. Hypothyroidism
133. A patient,35yrs old, who suffers of autoimmune gastritis and vitamin B12-deficiency anemia, presents signs of
funicular myelosis. Data of CBC: erythrocyte-2,2.1012/l, Hb-80g/l, LC-2,4.109/l, ESR-40mm/hr, direct bilirubin-
4,6micmol/l, indirect bilirubin-27micmol/l. What is funicular myelosis?

a. demyelinization of posteriolateral columns of spinal cord

b. demyelinization of anterior columns of spinal cord

c. affectionofbraincortex

d. demyelinization of peripheral nerves of lower limbs

e. nocorrectanswer

134. A 77-year-old woman is admitted to the hospital because of difficulty walking. She has had progressive pain
and paresthesia of bothfeet over the past 3 weeks. She has a history of mild hypertension treated with
hydrochlorothiazide and hypothyroidism treated withthyroid replacement therapy. Her pulse is 80/min, respirations
are 16/min, and blood pressure is 150/80 mm Hg. Neurologic

examination shows decreased ankle jerk reflexes bilaterally and decreased vibratory sense and proprioception in the
lowerextremities. Laboratory studies show:
Hemoglobin 10 g/dL.Leukocyte count 11,000/mm3 with a normal differential.Mean corpuscular volume 106 μm3

Which of the following is the most likely diagnosis?

22
(A) Diabetic neuropathy

(B) Hypothyroidism

(C) Pulmonary osteoarthropathy

(D) Spinal stenosis

(E) Vitamin B12 (cobalamin) deficiency

135. A 21-year-old woman comes to the physician for counseling prior to conception. She delivered a female
newborn with anencephaly 1 year ago. The newborn died at the age of 4 days. She asks the physician if she can
take any vitamins to decrease her risk for conceiving a fetus with anencephaly. It is most appropriate for the
physician to recommend which of the following vitamins?

a) Vitamin B12 (cyanocobalamin


b) Folic acid
c) Vitamin B1 (thiamine)
d) Vitamin B2 (riboflavin)
e) Vitamin B6 (pyridoxine)

136. Pernicious anemia and folic acid deficiencies are example of:

A. Microcytic anemias

b. Megaloblasticvanemias

C. Normochromic, normocytic anemias

D. Aplastic anemias

137. Hypersegmented, macrocytic neutrophils are often noted in the following condition:

A. IDA

B. Pernicious anemia

C. Acute myelogenuos leukemia

138. A patient,37yrs old, who suffers of autoimmune gastritis and vitamin B12-deficiency anemia, presents signs of
funicular myelosis. Data of CBC: erythrocyte-2,2.1012 /l, Hb-80g/l, LC-2,4.109/l, ESR-40mm/hr, direct bilirubin-
4,6micmol/l, indirect bilirubin-27micmol/l. What is probable reason of funicular myelosis?
a. prolongedhypoxiaofnervoussystem

b. increased bilirubin content in blood serum

c. * accumulation of propionic and methylmalonic acids in blood serum

d. infectioncontaminationduetoleukaemia

e. malnutritionduetogastritis

23
139.A patient,35yrs old, who suffers of autoimmune gastritis and vitamin B12-deficiency anemia, presents
signs of funicular myelosis. Data of CBC: erythrocyte-2,2.1012 /l, Hb-80g/l, LC-2,4.109/l, ESR-40mm/hr, direct
bilirubin-4,6micmol/l, indirect bilirubin-27micmol/l. Which substance is not produced with patient’s gastric
mucosa responsible for vitamin B12 absorption in intestine?
a. fullhydrochloricacid

b. combinedhydrochloricacid

c. pepsin

d. * gastromucoprotein

e. lacticacid

140. A patient, 60yrs old, complains of general weakness, fatigue, parestesia in limbs. He had resection of stomach
3years ago because of peptic ulcer. Data of examination: the tongue is of raspberry colour and smooth, the patient
is not stable in Romberg’s position. The patient is diagnosed vitamin B12 deficiency anemia. How do you explain
parestesia?

a. affectionofcentralnervoussystem

b. * affectionofperipheralnervoussystem

c. dystensionofskin

d. affectionofsubcutaneousfat

e. allmentioned

141. Patient I. 40 years old, 5 years ago was undergone the resection of stomach because of peptic ulcer
complication. The expressed general weakness, shortness of breath, appeared lately. Blood test: er.
3,1x1012/l, Hb 60 gr/l, colorindex 0,6, leukocytes 4,5x109/l, eosynophiles 2%, stub 3%, segm. neutr. 55%,
lymph. 32%, mon. 8%, ESR 5 mm/hour. What laboratory test will help in clarification of diagnosis?

a. levelofilirubin

b. levelofglucose

c. * levelofserumiron

d. levelofcreatinine

e. levelofamylase

142) Which of the following is the characteristic feature of anemia caused by the folic acid deficiency?
a) Microcytic anemia
b) Megaloblastic anemia
c) Hypochromic anemia
d) None of the above

41 ВСЕГО

24
ГАСТРО

143. Which is the most common cause of peptic ulcer disease (PUD) of the following:
a) smoking;
b) NSAID’s;
c) Zollinger-Ellison syndrome;
d) ethanol excess;
144. Functions of kidney:
a) excretory, regulatory, endocrine, metabolic;
b) protein inhibitor, circulation, transportation, excretory;
c) protein produced, regulatory, excretory;
d) excretory, transportation, metabolic;

145. A patient on your ward is diagnosed with hepatocellular carcinoma. You


are asked to perform a tumour marker level on this patient. Which of the following
tumour markers are elevated in hepatocellular carcinoma?
A. α-fetoprotein
B. Carcinoembryonic antigen (CEA)
C. CA 15-3
D. HcG
E. CA 125

146. A 47-year-old woman presents to your clinic with a three-month history of dysphagia. There is no
history of drastic weight loss and the patient experiences symptoms when swallowing solids but not liquids.
Which of the following is not an obstructive cause of dysphagia?

A. Pharyngeal carcinoma

B. Oesophageal web

C. Achalasia

D. Peptic stricture

E. Retrosternal goitre

147. You see a 47-year-old man in clinic with a three-month history of epigastric dull abdominal pain. He
states that the pain is worse in the mornings and is relieved after meals. On direct questioning, there is no history of
weight loss and the patient’s bowel habits are normal. On examination, his abdomen is soft and experiences
moderate discomfort on palpation of the epigastric region. The most likely diagnosis is:

A. Gastritis

B. Gastro-oesophageal reflux disease (GORD)

C. Duodenal ulcer

D. Gastric carcinoma

148. A 55-year-old woman is referred by her GP for upper gastrointestinal (GI) endoscopy

following a four-month history of epigastric pain despite treatment with antacids

and proton pump inhibitors (PPIs). The results demonstrate a duodenal ulcer

The most appropriate treatment is:


A. Seven-day course of twice daily omeprazole 20 mg, 1 g amoxicillin and 500 mg clarithromycin

B. Seven-day course of twice daily omeprazole 20 mg

C. Seven-day course of twice daily omeprazole 20 mg and 1 g amoxicillin

D. Seven-day course of twice daily omeprazole 20 mg and 500 mg clarithromycin

25
E. Seven-day course of twice daily 1 g amoxicillin and 500 mg clarithromycin

149.Which of the following is the most common cause of duodenal ulcers?

A. NSAIDs

B. Helicobacter pylori

C. Alcohol abuse

D. Chronic corticosteroid therapy

E. Zollinger–Ellison syndrome

150.A 56-year-old man presents with a 2-week history of diarrhoea which has not

settled following an episode of ‘food poisoning’. Which of the following would

be the most appropriate investigation?


A. Full blood count

B. Urea and electrolytes

C. Stool sample for microscopy, culture and sensitivities

D. Abdominal x-ray

E. Liver function tests

151. You see a 75-year-old man with an acute episode of haematemesis, who was

admitted the night before and is awaiting an upper GI endoscopy. You are asked on

the ward round about the common causes of upper GI bleeding. From the list below,

which of the following is the most common cause of upper GI bleeding?


A. Mallory–Weiss tear

B. Peptic ulcers

C. Oesophageal varices

D. Drug induced

E. Malignancy

152. A 67-year-old man presents feeling unwell and complaining of general malaise. He

mentions a long history of alcohol abuse and his past medical history shows

deranged liver function tests. Which of the following clinical signs does not form

part of chronic liver disease?

A. Finger clubbing

B. Palmer erythema

C. Spider naevia

D. Koilonychia

E. Jaundice

153. A 47-year-old man presents complaining of weight gain, on examination there

is an abdominal distension with a fluid thrill. Which of following is not a cause of

26
ascites secondary to venous hypertension?

A. Congestive heart failure

B. Cirrhosis

C. Constrictive pericarditis

D. Budd–Chiari syndrome

E. Nephrotic syndrome

154. A 35 year old woman who was born in Vietnam has had her dyspepsia diagnosed by endoscopy as a duodenal
ulcer. Her pathology is positive for Helicobacter pylori. She has not previously been diagnosed with peptic ulcer
disease. What would be the preferred treatment option?

a) Combination antibiotics plus omeprazole

b) Omeprazole alone

c) Surgical vagotomy and pyloroplasty

d) Octreotide

e) Ranitidiine

155. A 35-year-old white male presents with dyspepsia. He has had no symptoms that suggest gastroesophageal
reflux or bleeding, but a test for Helicobacter pylori is positive. He was treated with omeprazole (Prilosec),
amoxicillin, and clarithromycin (Biaxin) for 2 weeks. On the follow-up visit, he is asymptomatic.

Which one of the following is recommended to test for the eradication of H. pylori in this patient?

a) Immunoglobulin G serology

b) A urea breath test

c) Upper endoscopy with a biopsy

d) An upper gastrointestinal series

156. A 38 year old woman presents complaining of epigastric pain that is exacerbated by eating. An EGD reveals
the presence of a duodenal ulcer. A test for Helicobacter pylori is positive for the H. pylori. Which of the following
is the most appropriate treatment?

a) Metronidazole

b) Lansoprazole

c) Clarithromycin and Amoxicillin

d) Metronidazole and amoxicillin

e) Lansoprazole and clarithromycin and amoxicillin

157. A 40-year-old white male has elevated transaminase levels at a routine visit, but your clinical evaluation shows
no apparent cause. At a follow-up visit 6 months later they remain elevated. He remains asymptomatic and has a
normal physical examination. You order further tests, including serologic blood analyses.

Which one of the following imaging studies would be most appropriate?

a) Abdominal ultrasonography

b) CT with contrast

c) MRI of the abdomen

d) Magnetic resonance angiography (MRA) of the abdomen

27
e) Positron emission tomography (PET) of the abdomen

158. A 57-year-old male has laboratory screening performed prior to his annual examination. One of his six fecal
occult blood tests is positive. A CBC, chemistry panel, lipid panel, and prostate-specific antigen level are all normal.

Which one of the following would be the most appropriate course of action?

a) Repeat the fecal occult blood testing in 3 months

b) Perform a rectal examination in the office, and if a stool guaiac is negative repeat the fecal occult blood testing in
3 months

c) Refer for colonoscopy

d) Refer for flexible sigmoidoscopy

159. A 65-year-old male smoker complains of dyspepsia, weight loss, early satiety, and occasional nausea and
vomiting. Which one of the following would be the initial diagnostic method of choice?

a) Upper gastrointestinal endoscopy

b) CT of the upper abdomen

c) A single contrast upper GI barium swallow

d) Endoscopic ultrasonography

160 A 34-year-old man reports a 1-day history of hematemesis. He feels well, but does describe occasional
abdominal discomfort. He denies alcohol use. On examination, his abdomen is slightly tender without peritoneal
signs. His stool is not bloody, but his fecal test for occult blood is positive.
Which of the following is the most appropriate next step?
a. Gastric lavage
b. Barium study
c. Endoscopy
d. Red cell scan
e. Angiography

161. A 66-year-old woman comes to the physician with a 4-week history of abdominal symptoms. She complainsof
upper abdominal discomfort and occasional dull epigastricpain accompanied by nausea. Her symptomsare often
worse after eating, especially with large meals. Which of the following is the most appropriate next step in
management of this patient?
A. Abdominal ultrasound
B. Antacids and metoclopramide
C. Helicobacter pylorystool antigen testing
D. Upper gastrointestinal endoscopy

162. A 67-year-old man complains of episodic abdominal pain,indigestion, “heartburn,” dysphagia,.The pain is
concentrated in the epigastrium and is gnawing in quality. It wakes him up during the night and is promptly relieved
by a glass of water and a piece ofbread. He denies vomiting or diarrhea, but has experienced an occasional "dark
stool." His vital signs are with in normal limits. Physical examination shows mildepigastricdiscomfort on deep
palpation.Esophagogastroduodenoscopy reveals peptic ulcer

Which of the following is the best option to provide long-term symptom relief in this patient?
A. Four weeks of omeprazole
B. Antibiotics and pantoprazole
C. NSAIDs as needed
D. Cholecystectomy

163. A person js HBsAg positive, but Anti- HBc Ab is nega ¬ tive. What should be the next step?
a. Repeat test after 6 months
b. Check HBeAg, if positive start interferon
c. Check HBV DNA load

28
d. Reassure patient that he does not have any disease

164. A patient has anti HBs without any other without any other antigen or antibody against HBV.This indicates:
a. Vaccinated
b. Chronic infection
c. Persistent carrier
d. Acute infection

165. A 74-year-old man complaining of bitter taste and substernal burning 30-40 minutes after
meals. The burning is relieved with antacids and worsened by lying supine.He has had an unintentional weight loss
of 4.5 kg (1 0 lb}over the past 3 months. . Chest x-ray andelectrocardiogram show no abnormalities.
Which of the following is themost appropriate next step in management of this patient?
A.Upper gastrointestinal endoscopy
B. Esophageal pH monitoring
C. He/icobacterpylori stool antigen testing
D. Trial of famotidine

166. Most common route of transmission of hepatitis C:


a. I.V. drug abuse
b. Sexual contact
c. Factor 8 concentrate
d. Feco-oral route

167. Incubation period of hepatitis B is:


a. 6 weeks to 6 months
b. 6 days to 6 weeks
c. 6 months to 6 years
d. More than 6 years

168. Co-infection is essential for disease presentation in:


a. Hepatitis A
b. Hepatitis B
c. Non-A Non-B hepatitis
d. Delta hepatitis

169.Definitive diagnosis of acute pancreatitis is done by-


a) Amylase, Lipase
c) S. alkaline phosphatase d) Increased Ca
a) Hyperglycemia

170.Raised serum amylase levels are used to diagnose -


a) Autoimmune disease b) Degenerative disease
c) Acute cholecystitis d) Acute pancreatitis

171. Patient comes for blood donation but he has Hbs Ag and HbeAg positive, and serum transminases level is
normal. What would be the next line of management:
a . Treat with interferon
b. HBV DNA estimation
c. Liver biopsy
d. Observation

172. Which one of the following markers in the blood is the most reliable indicator of recent hepatitis B infection ?
a. HbsAg
b. IgG anti- HBs
c. IgM anti - HBc
d. IgM anti - Hbe

173. Which statement is wrong regarding Hepatitis B?


a. It is due to RNA virus
b. Blood is the main source of infection

29
c. Chronicity is present
d. It may turn into hepatocellular carcinoma

174. The commonest hepatotropic virus progressing tochronicity is :


a. HEV
b. HAV
c. HCV
175. A 69-year-old female presents to her GP with a two month history of indigestion. She is otherwise well, has not
had a similar episode before and takes no regular medication. Of note there is no recent weight loss or vomiting and
abdominal examination is unremarkable. What is the most appropriate initial management?
A. Long-term course of a H2 receptor antagonist
B. Urgent referral for endoscopy
C. One month course of a full-dose proton pump inhibitor
D. Urea breath testing and treat for H pylori if positive

176. Hepatitis B infectivity is indicated by:


a. Anti-HBsAg
b. HBsAg + HBeAg
c. Anti-HBsAg - Anti-HBc
d. Anti-HBeAg + Anti-Hbs Ag

177. A 69 year old smoker has had increasing dysphagia when eating solid food which has been on going for the
past 3 months. He has notice a drop of 8 kg in weight in the past few months. What SINGLE investigations is most
likely to lead to a diagnosis?

A. Barium swallow

B. Chest X-ray

C. Computed tomography chest

D. Endoscopy and biopsy

E. Videofluoroscopy

178. All of the following are noticed in cirrhosis of liver, except:

a. Raised serum albumin

b. Excessive urobilinogenuria

c. Prolonged prothrombin time

d. Raised serum globulin

179 . A man presents with history of hemetemesis of about 500 ml of blood . On examination , spleen is palpable 5
cms below the left costal margin . The most likely

diagnosis is:

a. Portal Hypertension

b. Gastric ulcer

c. Drug induced

d . Hepatitis A

180. Portacaval Encephalopathy is treated with:

a. Lactulose

b. Large amounts of proteins

30
c. Emergency portal systemic shunt surgery

d. Diuretics

181. Earliest sign in hepatic encephalopathy is:

a. Asterixis

b. Alternate constriction and dilated pupil

c. Constructional apraxia

d. Psychiatric abnormalities

182. A person js HBsAg positive, but Anti- HBc Ab is negative. What should be the next step?

a. Repeat test after 6 months

b. Check HBeAg, if positive start interferon

c. Check HBV DNA load

d. Reassure patient that he does not have any disease

183. Drug depended ulcer is induced:

a. aspirin; metoprolol;

b. metoprolol; prednisolon;

c. prednisolon; aspirin;

d. salmeterol; streptokinase.

184. Pain in epigastrium after meal is symptom ulcer disease of :

a. stomach;

b. distal part of esophagus;

c. duodenum.

185. Treatment of drug-induced gastritis:

a. Mesoprostol

b. H2 receptor blockers

c. Antacids

d. Aspirin

186. Indications of colonscopy:

a. rectal bleeding, anemia, suspected inflammatory bowel disease

b. suspected inflammatory bowel disease, severe shock.

c. severe respiratory disease

d. cardiac arrhythmia.

187. Contraindications of colonoscopy:

a. rectal bleeding, anemia, suspected inflammatory bowel disease

b. suspected inflammatory bowel disease,

c. severe respiratory disease, severe shock.

31
d. suspected inflammatory bowel disease

188. Ascites in liver cirrhosis is formed as a result of:

a. secondary hyperaldosteronism, hypoalbuminemia, Portal Hypertension

b. hypoaldosteronism, potassium loss, salt retention

c. hypertension, salt retention

d. all the above

189. Decrease acid-secretory function occurs more frequently:

a. in chronic superficial gastritis

b. in chronic gastritis, antrum

c. in chronic atrophic gastritis

d. with hypertrophic gastritis

190. The method of diagnosing the cause and localization of bleeding from the upper gastrointestinal tract is:

a. Radiography

b. Portomanometriya

c. Laparoscopy

d. gastroduodenoscopy

191. To exclude the malignancy of gastric ulcer should be used:

a. X-ray examination

b. endoscopy

c. stool for occult blood

d. endoscopy with biopsy

192. Chronic atrophic gastritis is caused by:

a. Antibody response to the G cells of the stomach, which reduces the production of gastrin leads to achlorhydria;

b. mucosal atrophy in patients with atherosclerosis;

c. long-term use of glucocorticoids;

d. NSAIDs.

193. Complications of duodenal ulcer;

a. Bleeding;

b. portal hypertension;

c. jaundice;

d. ascites;

194. Which of the following is not an indication for cholecystectomy:

32
a. 70 year old male with symptomatic gall stone

b. 20 years old male with sickle cell anemia and

symptomatic gallstones

c. 65 year old female with a large gallbladder polyp

d. 55 year old with an asymptomatic gallstone

195. A 27-year-old woman presents with a 9-month history of bloody diarrhea and crampy abdominal pain. Three
weeks ago, she noticed that her left knee was swollen, red, and painful. Her temperature is 38°C (101°F),
respirations are 32 per minute, and blood pressure is 130/90 mm Hg. Abdominal palpation reveals tenderness over
the left lower quadrant. Laboratory studies show moderate anemia, with a hemoglobin level of 9.3 g/dL.
Microscopic examination of the stool reveals numerous red and white blood cells. A diffusely red, bleeding,
friable colonic mucosa is visualized by colonoscopy. The colon is subsequently removed and the surgical
specimen is shown in the image. Which of the following is the most likely diagnosis?

a) Adenocarcinoma
b) Ulcerative colitis
c) Carcinoid tumor
d) Crohn disease
e) Pseudomembranous colitis

196. Characteristic of pain in chronic gastritis:

a. localisation in mesogastriumbeginnig in night ;

b. localisation in right hypochandrium with irradiation in right shoulder;

c. localisation in epigastrium which is began after meal;

d. localisation in epigastrium which is redused after meal;

e. localisation in epigastrium, which is indused fat meal.

197. Most common cause of chronic pancreatitis

a) Alcoholism, Gallstone
b) Ulcerative colitis

c) Tropical pancreatitis
d) Severe protein-calorie malnutrition

198. What percentage of heavy drinkers ultimately develops chronic pancreatitis?

a) 10-20%
b) 40-75%
c) 80-90%
d) 98%

199. Which complication would you expect from a person suffering from chronic pancreatitis?

a. Diabetes mellitus, leading to elevated levels of blood glucose


b. Hyperthyroidism, leading to increased weight loss
c. Heart failure, leading to a smaller heart size
d. Retinal degeneration, leading to better visual acuity

33
200. 2. A 21-year-old man is brought to the emergency room with symptoms of acute intestinal obstruction. His
temperature is
38°C (101°F), respirations are 25 per minute, and blood pressure is 120/80 mm Hg. Physical examination
reveals a mass in the right lower abdominal quadrant. The patient subsequently undergoes surgery, and a
segmental lesion involving the terminal ileum is resected (shown in the image). Which of the following is the
most likely diagnosis?

a) Adenocarcinoma
b) Carcinoid tumor
c) Crohn disease
d) Pseudomembranous colitis
e) Ulcerative colitis

201. Omeprazole belongs to the group of drugs:

a. blockers;

b. M-holinoblokatory;

c. blockers of histamine H1-receptor antagonists;

d.proton pump blockers.

202. Etiological treatment with interferon in chronic viral hepatitis are beginning to phase:

a. Replication

b. integration

c. proliferation

d. fibrosis

203. Which is characteristic of cytolytic syndrome?

a. increase: ALT, AST, LDG5

b. increase: direct bilirubin, alkaline phosphatase, uGTP, cholesterol

c. The increase in indirect bilirubin, ALT decrease, uGTP

204. Which is typical for mesenchimal inflammatory syndrome?

a. reduction: albumin, prothrombin, transferrin, cholesterol

b. increase: gammaglobulin, ESR, thymol, CRP

c. increase: ALT, AST, LDG5

205. Crohn’s disease is associated with which of the following?

A) Inflammation limited to the superficial layer of the bowel wall

B) The affinity to involve the rectosigmoid junction

C) Decreased risk of colon cancer

D) Continuous mucosal areas of ulceration that affect the anus

E) Fistula formation

34
206. HBs Ag positive person may have all of the following associated Renal lesions, Except:

a. Membranous Glomerulonephritis (MGN)

b. Membrano proliferative Glomerulnephritis (MPGN)

c. Mesangiocapillary Glomerulonephritis

d. Focal Segmental Glomerulosclerosis (FSGS)

207. None invasive diagnostic methods for H.pylori verification are:

a. histologycal;

b. ureas test;

c. microbiological blood test on H. Pylori;

d. microbiological stool test on H. Pylori;

208. Acidity in gastritis B is:

a. normal;

b. high;

c. lower than the normal.

209. Main clinical symptom in duodenum ulcer disease is:

a. pain on empty stomach;

b. diarrhoea;

c. eructation;

d. early pain after meal;

210. First line drugs for H. Pylori eradication are:

a. amoxicillin, clarithromicin;

c. metronidazole; furazolidone.

d. sumamed;

211. 32 y.o female compliance diarrhea, vomiting, nausea, abdominal pain, weakness, mody change;On
examination: Her blood pressure 120/70mm.hg, RR-25, HR-90/min, the puls is regular; On chest examination :
normal breath and heart sound; on palpation of the abdomen painless and tenderness. CBC is normal range, blood
occult test is negative
What is your suspected diagnosis?
a. Peptic ulcer disease
b. GERD and chronic enteritis
c. Irritable bowel syndrome
d.Ulcerative colitis

35
212. Typical localization of chronic gastritis type A is:

a. antral part of the stomach;

b. gastroesophageal part of the stomach;

c. cardia;

d. body of the stomach;

213. Clinical manifestations of chronic gastritis:

a) cytolytic syndrome;

b) cholestatic syndrome;

c) syndrome of portal hypertension;

d) syndrome of gastric dyspepsia.

214. Which is not a complication of hepatic cirrhosis:

a) spontaneous bacterial peritonitis;

b) hepatocellular carcinoma;

c) thrombocytosis;

d) portal hypertension;

e) portal vein thrombosis.

215. Which of the interferons has mainly antiviral activity?

1) γ-IFN

2) α-IFN

3) β-IFN

216. A 26-year-old man presents with dyspepsia. No alarm symptoms are present. This is his first episode and he
has no significant medical history of note. A test-and-treat strategy is agreed upon. What is the most appropriate
investigation to test for Helicobacter pylori?

A. Upper gastrointestinal endoscopy

B. Esophageal pH monitoring

C. Stool culture

D. Urea breath testing

217. A 27-year-old man complains of episodic abdominal pain. The pain is concentrated in the epigastrium and is
gnawing in quality. It wakes him up during the night and is promptly relieved by a glass of water and a piece
ofbread. He denies vomiting or diarrhea, but has experienced an occasional "dark stool." His vital signs are with
in normal limits. Physical examination shows mildepigastric discomfort on deep palpation. What is the most
likely diagnosis?

36
a) chronic gastritis В

b) chronic gastritis А

c) gastric ulcerdisease

d) duodenal ulcer disease

218. A 60-year-old woman comes to the physician with a 4-week history of abdominal symptoms. She complainsof
upper abdominal discomfort and occasional dull epigastric pain accompanied by nausea. Her symptoms are often
worse after eating, especially with large meals. Which of the following is the most appropriate next step in
management of this patient?

A. Abdominal ultrasound

B. Antacids and metoclopramide

C. Helicobacter pylorystool antigen testing

D. Esophagogastroduodenoscopy

219. A 44-year-old white male comes to the office and says, "I have had this persistent, upper abdominal pain for
the past 2 months. Physical examination reveals mild tenderness in the epigastric region.

Esophagogastroduodenoscopy reveals peptic ulcer

What is the best next step in the management of this patient?

A. Radical gastrectomy with en bloc resection of spleen and regional nodes

B. Give a combination of omeprazole, clarithromycin, and amoxicillin.

C. Give combination chemotherapy.

D. Radiotherapy

~E. Observation and strict surveillance

220. A young man presents with HBsAg positive anti HBcIgM positive, HBeAg negative and normal
levels of AST and ALT. He is asymptomatic. What is the next line of management.

a) Wait and Watch b) Lamivudin

c) Immunoglobulin d) Liver Transplant

221. All the following hepatitis


viruses is transmitted by parenteral route except- a) Hepatitis В virus b)
Hepatitis С virus

c) Hepatitis D virus d) Hepatitis E virus

37
222. A 52-year-old man presents to your office after passing a black stool. He also describes occasional abdominal
discomfort and nausea but denies hematemesis. He says that food seems to help his abdominal pain, so he eats
frequently during the day and keeps some snacks on his night stand. As a consequence he has gained 5 pounds over
the last year. The fecal occult blood test is positive. Which of the following is the most likely cause of his
condition?

A. Gastritis

B.Peptic ulcer disease

C. Inflammatory bowel disease

D. Hepatitis B

223. A 56-year-old man is brought to the emergency department with lethargy and confusion. He has a history
ofcirrhosis secondary to chronic alcoholism. He is taking spironolactone and furosemide.

Onphysical examination, he has a flapping tremor of his hands when they are held out. Abdominal examination
shows distension with shifting dullness anddiffuse tenderness to palpation. There is bilateral pitting edema of his
lower extremities.

Which of the following is the most appropriate next step in management?

A. Esophageal pH monitoring

B. Upper gastrointestinal endoscopy

C.Diagnostic paracentesis

D. He/icobacter pylori stool antigen testing

224. A 54-year-old man complaining of bitter taste and substernal burning 30-40 minutes after

meals. The burning is relieved with antacids and worsened by lying supine.He has had an unintentional weight loss
of 4.5 kg (1 0 lb}over the past 3 months. . Chest x-ray and electrocardiogram show no abnormalities.

Which of the following is themost appropriate next step in management of this patient?

A.Upper gastrointestinal endoscopy


B. Esophageal pH monitoring

C. He/icobacter pylori stool antigen testing

D. Trial of famotidine

225. You wish to screen a patient for hepatitis B infection. Which one of the following is the most suitable test to
perform?

A. HBcAg

B. HBsAg

C. Hepatitis B viral load

D. anti-HBs

38
226. A 59-year-old female presents to her GP with a two month history of indigestion. She is otherwise well, has not
had a similar episode before and takes no regular medication. Of note there is no recent weight loss or vomiting and
abdominal examination is unremarkable. What is the most appropriate initial management?

E. Long-term course of a H2 receptor antagonist

F. Urgent referral for endoscopy

G. One month course of a full-dose proton pump inhibitor

H. Urea breath testing and treat for H pylori if positive

227. Which one of the following statements regarding hepatitis C is correct?

A. Cannot be transmitted vertically from mother to child

B. Interferon-alpha and ribavirin are the treatments of choice

C. It is more infectious than hepatitis B following a needle stick injury

D. Breast feeding is contraindicated in mothers with hepatitis C

228. A 45-year-old man with a long history of alcohol intake comes into the emergency room with

upper gastrointestinal (UGI) bleeding. Urgentendoscopy reveals - the following findings the esophageal folds are
thick and tortuous, giving rise to a wormy or worm-eaten appearance.

Which of the following is the most likely diagnosis?

(A) esophageal varices

(B) esophageal carcinoma

(C) foreign body

(D) tertiary waves

(E) Barrett’sesophagus

229. A young patient presenting with massive hematemesis was found to have splenomegaly. In this case
the most likely source of bleeding is -

a) Duodenal ulcer b) Esophageal varices, c)Erosive mucosal disease d) Gastric ulcer

230. A 26-year-old doctoral candidate comes to the office 1 day after she passed bloody mucus with her stool. She
noticed mild abdominal cramping and some constipation preceding the episode; she has not had diarrhea or
melanotic stools. She has not had any exposure to unusual or poorly kept food or to any known infectious agents.
Her only medication is acetaminophen for occasional headaches. Her father had colon cancer and died at age 48
years from metastatic disease. She recalls being told that an aunt died of an unknown abdominal tumor. The patient's
vital signs are temperature 37.6°C (99.6°F), pulse 90/min, respirations 18/min, and blood pressure 110/74 mm Hg.
Abdomen is mildly distended, nontympanitic, and tender to palpation only in the right lower quadrant. Pelvic
examination shows no masses or mucosal lesions. Which of the following is the most appropriate diagnostic study?
A) Colonoscopy
B) Esophagogastroduodenoscopy
C) Lower gastrointestinal barium study
D) Ultrasonography of the appendix
E) Upper gastrointestinal barium study with small-bowel follow-through

231. In hepatic cirrhosis, which of the following is increased - a) Alpha 1 globulin b) Alpha 2 globulin

c) Gamma globulin d) All of the above

232. A 40 year old patient, a known case of cirrhosis develops acute episode of GI bleed. Initial therapy
given for 6 hours.Which of the following procedure is useful -

39
a)Nasogastric aspiration b) Urgent endoscopy , c)Sedation d) Ultrasound

233. A young boy, R. presents with massive hemetemesis. He had fever for 15 days few days back which was
treated. Clinical examination reveal moderate splenomegaly No other history is positive Probable diagnosis is -
a) Drug induced gastritis b) Oesophageal tear, b) Bleeding duodenal ulcer d) Oesophageal varices

234. The complication of liver cirrhosis

1)esophageal bleeding

2)perforation of the stomach

3)penetration

4) pyloric stenosis

235. +Which of the following is the most common cause of ulcerative colitis-related mortality?
a) Colonic adenocarcinoma
b) Toxic megacolon
c) Perforated colon
d) Colonic infarction

236. First line of treatment in severe ascites – a)Bed rest and salt restriction, b)Heavy peracentesis

c)Shunt, d)Large dose of diuretics

237. A 40 year old patient, a known case of cirrhosis develops acute episode of GI bleed. Initial therapy
given for 6 hours.Which of the following procedure is useful -

a) Nasogastric aspiration b) Urgent endoscopy , c)Sedation d) Ultrasound

238. +Which of the following is the preferred therapy in mild ulcerative colitis confined to the rectum?

a) Topical mesalazine given by suppository


b) Oral prednisone
c) Intravenous azathioprine
d) Hydrocortisone given by enema

239. Pain in epigastrium after meal is symptom ulcer disease of :a. stomach;

b. distal part of esophagus; c. duodenum.

240. Main clinical symptom in duodenum ulcer disease is: a. pain on empty stomach; b. diarrhoea; c. eructation;
d. early pain after meal; e. nausea

241. A 27 year old male is is incidentally HBsAg positive. DNA-PCR for hepatitis В revealed 1000 copies/ml. The
patient is suffering from?

a) Active HBV carrier b) Acute hepatitis Вс) Chronic hepatitis Вd) Inactive HBV carrier

242. +A 27-year-old woman presents with a 9-month history of bloody diarrhea and crampy abdominal
pain. Three weeks ago, she noticed that her left knee was swollen, red, and painful. Her temperature is 38°C
(101°F), respirations are 32 per minute, and blood pressure is 130/90 mm Hg. Abdominal palpation reveals
tenderness over the left lower quadrant. Laboratory studies show moderate anemia, with a hemoglobin level of 9.3
g/dL. Microscopic examination of the stool reveals numerous red and white blood cells. A diffusely red, bleeding,
friable colonic mucosa is visualized by colonoscopy. The colon is subsequently removed and the surgical
specimen is shown in the image. Which of the following is the most likely diagnosis?

f) Adenocarcinoma

40
g) Ulcerative colitis
h) Carcinoid tumor
i) Crohn disease
j) Pseudomembranous colitis

243. + A 21-year-old man is brought to the emergency room with symptoms of acute intestinal
obstruction. His temperature is
38°C (101°F), respirations are 25 per minute, and blood pressure is 120/80 mm Hg. Physical examination
reveals a mass in the right lower abdominal quadrant. The patient subsequently undergoes surgery, and a
segmental lesion involving the terminal ileum is resected (shown in the image). Which of the following is the
most likely diagnosis?

f) Adenocarcinoma
g) Carcinoid tumor
h) Crohn disease
i) Pseudomembranous colitis
j) Ulcerative colitis

244. Chronic inflammatory bowel disease is associated with:


a) Chronic hepatitis
b) Fibrosis
c) Cholangiosarcoma
d) Primary sclerosing cholangitis
245. +Crohn’s disease is associated with which of the following?

A) Inflammation limited to the superficial layer of the bowel wall

B) The affinity to involve the rectosigmoid junction

C) Decreased risk of colon cancer

D) Continuous mucosal areas of ulceration that affect the anus

E) Fistula formation
246. A patient with H. pylori
infection is treated with drugs. The best method to
a) Rapid urease test detect presence of residual H. pylori infection in
b) Urea breath test this person is -
c) Endoscopy and biopsy
d) Serum anti H. pylori titre

247. +34 y.o female compliance diarrhea, vomiting, nausea,


abdominal pain, weakness, mody change;On examination: Her blood pressure
120/70mm.hg, RR-25, HR-90/min, the puls is regular; On chest examination :
normal breath and heart sound; on palpation of the abdomen painless and
tenderness. CBC is normal range, blood occult test is negative
What is your suspected diagnosis?
a. Peptic ulcer disease
b. GERD and chronic enteritis
c. Irritable bowel syndrome
d.Ulcerative colitis

248. 35yo male with epigastric discomfort has been given triple therapy. He has

now returned after 4wks of epigastric discomfort. What inv would you do for
him?
a) ECG

41
b) H pylori breath test
c) Endoscopy and biopsy
d) US
249. A 64yo alcoholic who has been dx with liver cirrhosis presents with a massive
ascites. What is the mechanism of fluid accumulation in a pt with liver disease?
a. Cirrhosis
b. Portal HTN
c. Hypoalbuminemia
d. Liver failure
e. Hepatic encephalopathy

250. A 34yo man has an intermittent epigastric pain for 3wks. It is worse by
food but helped by some tablets he obtained from the pharmacy. He had a
similar episode 3yrs ago and his doctor gave him a course of 3 types of tablets
at the time. What is the most appropriate next inv?
a. Abdomen US
b. Barium meal
c. Serum H.Pylori antibodies
d. Urea breath test
e. Upper GI endoscopy

251. A 66 yo man has an intermittent epigastric pain for 3wks. It is worse by


food but helped by some tablets he obtained from the pharmacy. He had a
similar episode 3yrs ago and his doctor gave him a course of 3 types of tablets
at the time. What is the most appropriate next inv?
a. Abdomen US
b. Barium meal
c. Serum H.Pylori antibodies
d. Urea breath test
e. Upper GI endoscopy

252. 65yo male with epigastric discomfort has been given triple therapy. He has
now returned after 4wks of epigastric discomfort. What inv would you do for
him?
a) ECG
b) H pylori breath test
c) Endoscopy and biopsy
d) US

253. 1. A 38-year-old male visits his GP complaining of intermittent abdominal pain. This is epigastric in nature
and radiates through to the back. It is more severe before meal times and relieved by eating. A C14 breath test is
arranged which comes back positive. What is themost likely diagnosis?

a. Duodenal ulcer

b. GORD

c. Barrett’s oesophagus

d. Achalasia

e. Gastric ulcer

254. + Crohn’s disease is associated with which of the following?

A) Inflammation limited to the superficial layer of the bowel wall

B) The affinity to involve the rectosigmoid junction

C) Decreased risk of colon cancer

D) Continuous mucosal areas of ulceration that affect the anus

E) Fistula formation

42
255. A 40-year-old obese woman complains of right upper quadrant pain after
eating, which radiates into her back. What is the most likely diagnosis?
A. Duodenal ulcer
B. Hepatoma
C. Cholecystitis
D. Pancreatitis
E. Metastatic adenocarcinoma to the liver

256. Skip granulomatous lesions are seen in :


a) Ulcerative colitis
b) Crohn's disease
c) Whipple's disease
d) Reiter's disease
257. Invariably involved site in ulcerative colitis:
a) Sigmoid colon
b) Transverse colon
c) Ileum
d) Rectum
258. Regional enteritis (Crohn’s Disease) is characterised by:
a) Transmural thickening;
b) Diffuse involvement;
c) Exacerbations and remissions;
d) Severe diarrhoea;
259.A 32-year-old woman has a history of chronic diarrhea and gallstones and now has rectovaginal fistula. Which
of the following is the most likely diagnosis?
a) Crohn disease
b) Ulcerat ive colitis
c) Systemic lupus erythematosus
d) Laxative abuse

260.A 46-year-old female presented to the GP with a change in bowel habit for the past 2 months. She is having
looser stools than normal with blood mixed in with the stools. There is no history of any abdominal pain. What
would be your first choice investigation?

a. Abdominal X-ray

a) b. Faecal occult blood


b) c. Abdominal CT
c) d. Ultrasound abdomen
d) e. Colonoscopy

261. You are told by your registrar that one of your inpatients has been diagnosed with

primary sclerosing cholangitis (PSC). Your registrar suspects that the patient may

have an associated condition. Primary sclerosing cholangitis is associated with


which of the following diseases?

A. Thyroid disease

B. Systemic sclerosis

C. Rheumatoid arthritis

D. Ulcerative colitis

E. Irritable bowel syndrome

43
262. A 32-year-old woman has a history of chronic diarrhea and gallstones and now has rectovaginal fistula.
Which of the following is the most likely diagnosis?

a. Crohn disease
b. Ulcerat ive colitis
c. Systemic lupus erythematosus
d. Laxative abuse

263. A 35-year-old woman has chronic crampy abdominal pain and intermittent constipation and diarrhea,
but no weight loss or gastrointestinal bleeding. Her abdominal pain is usually relieved with defection.
Colonoscopy and upper endoscopy with biopsies are normal, and stool cultures are negative.

Which of the following is the most likely diagnosis?


a) Infectious colitis
b) Irritable bowel syndrome
c) Crohn disease
d) Ulcerative colitis

264. A young girl presents with abdominal pain and a recent change in bowel habit, with passage of mucus in
stool. There is no associated blood in stool and symptoms are increased with stress. The most likely
diagnosis is -
a) Irritable bowel syndrome
b) Ulcerative colitis
c) Crohn’s disease
d) Amebiasis
265. A 40 year old male presents with chronic diarrhea for one year. On in Microscopiccolitis vestigation,
the findings obtained were crypt abcess, crypt atrophy, cryptitis and mucositis. Most likely
diagnosisis-
a) Crohn'sdisease
b) Ulcerativecolitis
c)
d) Collagenouscolitis
266. A 24 years old female college student complains of diffuse lower abdominal pain. It has been present for
approximately 5 months and gets better with defecating. The onset is associated with more frequent and
loose bowel movement ( 3- 5x/day). She denies blood or bloating. No weight loss or night time symptoms.
What is the most likely diagnosis?
a) Ulcerative colitis
b) Irritable bowel syndrome.
c) Crohn's disease.
d) Lactose intolerance.

267. A 28-year-old man undergoes a sigmoidoscopy for longstanding diarrhoea and


weight loss. On visualization of the rectum, the mucosa appears inflamed and

friable. A rectal biopsy is taken and the histology shows mucosal ulcers with

inflammatory infiltrate, crypt abscesses with goblet cell depletion. From the list of

answers below, which is the most likely diagnosis describing the histology report?

A. Crohn’s disease

B. Pseudomembranous colitis

C. Irritable bowel syndrome

D. Ulcerative colitis

E. No diagnosis – the report is inconclusive

44
268. A 24-year-old male was referred for a colonoscopy to investigate his symptoms of weight loss and diarrhoea.
The endoscopist thought that they could view skip lesions in the terminal ileum and took several biopsies. What is
the histology most likely to show?

a. Ulcerative colitis

b. Coeliac disease

c. Irritable bowel syndrome

d. Crohn’s disease

e. Bacterial enterocolitis

269. An elderly male patient with prior history of hematemesis is having hx of long term use of aspirin and other
drugs, now presents with severe epigastric pain, dysphagia
and vomiting. He was connected to vital monitors which were not reassuring.
What is the management?
a. Oral antacids
b. IV PPI
c. Oral PPI
d. Endoscopy
270. Which of the following tests is used to detect hepatitis B
infection during the “window period”?
A) Hepatitis B surface antigen
B) Hepatitis B surface antibody
C) Hepatitis B core antibody (IgM)
D) Hepatitis B e antigen
E) Hepatitis B antibody to the delta agent

271. A 50 year old male with alcoholic cirrhosis underwent endoscopic band ligation in view of upper
GI bleed from esophageal varices. One month later he had recurrence of bleeding. What will be most
appropriate next line management after stabilising the patient
A. Start beta blockers
A. Endoscopic band ligation
B. Оctreotide
C. Liver transplant
272. A 28-year-old woman is presented with abdominal pain for more than 6 months the frequency of
which has increased to 1 day per week for last 3months and pain is associated with defecation. There is hardening
of stools since her symptoms have started. She has noted some mucus in the stool and mild bloating, but has not
noted any blood in the stools. There is no fever, nocturnal diarrhea or weight loss. Abdominal examination is
unremarkable. Routine laboratory tests show a normal complete blood count and blood glucose. The most likely
diagnosis in this patient is:
A. Colon cancer.
B. Irritable bowel syndrome.
C. Inflammatory bowel disease.
D. Malabsorption syndrome.

273An 88-year-old white woman is taking naproxen for osteoarthritis. She has noticed mild epigastric discomfort
for several weeks, but has continued the naproxen because of improvement in joint symptoms. She suddenly
develops hematemesis and hypotension. (CHOOSE 1 DIAGNOSIS)
a. Gastric ulcer
b. Aortoenteric fistula
c. Mallory-Weiss tear
d. Esophageal varices
e. Hereditary hemorrhagic telangiectasia (HHT)

45
274. The best way to eradicate H. pylori is
a. Omeprazole 20 mg PO daily for 6 weeks
b. Ranitidine 300 mg PO qhs for 6 weeks
c. Omeprazole 20 mg BID, amoxicillin 1000 mg BID, clarithromycin 500 mg BID
for 14 days
d. Pepto-Bismol and metronidazole BID for 7 days
e. Sulcrafate 200 g QID for 6 weeks

275. Which one of the following patients should be promptly referred for endoscopy?
A. A 65-year-old man with new onset of epigastric pain and weight loss
B. A 32-year-old patient whose symptoms are not relieved with ranitidine
C. A 29-year-old H pylori-positive patient with dyspeptic symptoms
D. A 49-year-old woman with intermittent right-upper quadrant pain following
Meals

276. A 36-year-old cigarette smoker complains of epigastric pain, well localized, nonradiating, and described as
burning. The pain is partially relieved by eating. There is no weight loss. He has not used nonsteroidal
anti-inflammatory agents. The pain has gradually worsened over several months.
The most sensitive way to make a specific diagnosis is
a. Barium x-ray
b. Endoscopy
c. urea breath test
d. Serum gastrin

277. A 51-year-old man with a lengthy history of medication-dependent reflux esophagitis sees his physician for an
annual physical examination.
Laboratory tests reveal a blood gastrin level three times the upper limit of normal. His physician expresses concern
that the patient is at risk of developing atrophic gastritis.
Which of the following medications is this patient most likely taking?
(A) Aluminum hydroxide
(B) Bismuth
(C) Cimetidine
(D) Misoprostol
(E) Omeprazole

277. A 45 - year old cirrhotic patient presented with severe haematemesis. The management of choice is-
a) Whole blood transfusion is the best
b) Colloids are preferred over crystalloids
c) Normal saline infusion
d) IV fluid with diuretics

279. An 88-year-old white woman is taking naproxen for osteoarthritis. She has noticed mild epigastric discomfort
for several weeks, but has continued the naproxen because of improvement in joint symptoms. She suddenly
develops hematemesis and hypotension. (CHOOSE 1 DIAGNOSIS)
a. Gastric ulcer
b. Aortoenteric fistula
c. Mallory-Weiss tear
d. Esophageal varices
e. Hereditary hemorrhagic telangiectasia (HHT)

280. A 53-yea r-old obese man p resents with abdominal pain that is worse with food for the last 8 months. H e
describes the pain as a deep stabbin g sensation . He
has dark stools that are foul smelling . He says lately he is afraid of eating, feels full quickly, and has lost 10 pounds
in the last month. The patient has a full bottle
of esomeprazole in his pocket. He takes ibuprofen for knee pain . His hemoglobin is 9.3 g/d l (normal range is 14 to
17) .
What is the most likely diagnosis?

46
a. Peptic ulcer disease
b. Pancreatitis
c. Cholecystitis
d. Non - ulcer dyspepsia

281. A 55-year-old man with a history of coronary artery disease and alcoholism presents to the emergency
department complaining that he vomited bright red blood twice this morning. He denies previous episodes
of bleeding or abdominal pain. On examination, he is a malnourished man in acute distress. His blood
pressure is 90/50 mm Hg and his pulse is 110/min. His mucous membranes are dry and his sclera are
icteric. Abdominal examination reveals a distended abdomen with an enlarged, palpable spleen. Purplish
striae are seen around the umbilicus. On rectal examination, Iarge hemorrhoids are seen, but the stool is
negative for blood.
Which of the following is the most likely diagnosis?
/ A. Erosive gastritis
/ B. Esophageal varices
/ C. Infectious enteritis
/ D. Mallory Weiss tear
/ E. Peptic ulcer disease

282. Central nervous system manifestion in chronic renal failure are a result of all of the following except -
a) Hyperosmolarity b) Hypocalcemia c) Acidosis d) Hyponatremia

283. Which one of the following statement for a patient with Irritable bowel syndrome considered we should refer
the patient to specialist?
a) diarrhea alternating with constipation
b) rectal bleeding
c) who take ASPIRIN
d) none

284. A 28-year-old man undergoes a sigmoidoscopy for longstanding diarrhoea and


weight loss. On visualization of the rectum, the mucosa appears inflamed and

friable. A rectal biopsy is taken and the histology shows mucosal ulcers with

inflammatory infiltrate, crypt abscesses with goblet cell depletion. From the list of

answers below, which is the most likely diagnosis describing the histology report?
A. Crohn’s disease

B. Pseudomembranous colitis

C. Irritable bowel syndrome

D. Ulcerative colitis

E. No diagnosis – the report is inconclusive

285. Which of the following symptoms is not associated with Irritable bowel syndrome?
A. UrgencyMucous on stool
B. Meleana
C. Backache
D. Low mood
286. Which of the following is clinically diagnostic of Irritable bowel syndrome?
A. Improvement with defecation
B. Non flushable/pale stools
C. Blood mixed with stool
D. LIF pain
E. Rebound tenderness

47
287. Once other disease condition have been ruled out, a person can be considered for the diagnosis
of irritable bowel syndrome if the symptoms were present for the last
A. One week
B. One fortnight
C. One month
D. Three months
288. Which psychological treatment option for irritable bowel syndrome (IBS) is supported by the best
evidence?
a) Cognitive–behavioral psychotherapy
b) Support group meeting
c) A + B
d) none

289. You are asked to see a 29-year-old woman diagnosed with ulcerative

colitis 18 months ago. Over the last 4 days she has been experiencing slight

abdominal cramps, opening her bowels approximately 4–5 times a day and has

been passing small amounts of blood per rectum. The patient is alert and orientated

and on examination her pulse is 67, blood pressure 127/70, temperature 37.3°C

and her abdomen is soft with mild central tenderness. PR examination is nil of

note. Blood tests reveal haemoglobin of 13.5 g/dL and a CRP of 9 mg/L. The most

appropriate management plan for this patient is:


A. Admission to hospital for intravenous fluid therapy and steroids

B. Oral steroid therapy + oral 5-ASA + steroid enemas + discharge

C. Admission and refer to surgeons for further assessment

D. Oral steroid therapy and discharge home

E. Reassurance and discharge home with no treatment required

290. For gastric ulcer and 12 duodenal ulcer in elderly and senile people it is characteristic:
a) lack of symptoms; predisposition to malignancy;
b) more severe course;
c) ulcers of large and gigantic sizes;

291. Which of the following is an acceptable treatment for Helicobacter pylori infection?
A) Bismuth, amoxicillin, metronidazole, and omeprazole
B) TMP-sulfamethoxazole, sucralfate, and metronidazole
C) Omeprazole, clindamycin, and sucralfate
D) Docusate, tetracycline, and metronidazole
E) Ranitidine, metronidazole, and ampicillin
292. A65-year-old man with a long history of alcohol intake comes into the emergency room with
upper gastrointestinal (UGI) bleeding. Urgentendoscopy reveals - the following findings the esophageal folds are
thick and tortuous, giving rise to a wormy or worm-eaten appearance.
Which of the following is the most likely diagnosis?
(A) esophageal varices
(B) esophageal carcinoma
(C) foreign body
(D) Barrett’sesophagus

293. Interferon treatment is recommended in chronic hepatitis B in patients with:

48
a. > HBV DNA and Normal ALT

49
b. > HBV DNA and > ALT
c. > HBV DNA and compensated cirrhosis
d. > HBV DNA and decompensated cirrhosis
294. A 27-year-old woman with chronic left iliac fossa pain and alternating bowel habit is diagnosed with irritable
bowel syndrome. Initial treatment is tried with a combination of antispasmodics, laxatives and anti-motility agents.
Unfortunately after 6 months there has been no significant improvement in her symptoms. According to recent
NICE guidelines, what is the most appropriate next step?
A. Low-dose tricyclic antidepressant
B. Cognitive behavioural therapy
C. Refer for sigmoidoscopy
D. Trial of probiotics

295. A 29-year-old man is reviewed. Four weeks ago he presented with a one month history of bloody
diarrhoea. He was previously fit and well prior to this episode. When initially reviewed he was passing on average
four loose stools a day with some visible blood. Colonoscopy showed extensive inflammatory changes consistent
with ulcerative colitis. He was started on oral mesalazine and a review appointment was made for today. He is still
passing around four bloody stools a day although he remains systemically well. What is the most appropriate course
of action?
A. Add oral prednisolone
B. Stop oral mesalazine and start oral prednisolone
C. Rectal corticosteroids
D. Admit for intravenous corticosteroids
E. Add oral azathioprine

154

Renal

294. Anemia that is seen in patients with chronic renal disease is usually caused by insufficient
A) Iron stores
B) Vitamin B12
C) Renin levels
D) Erythropoietin levels
E) Folate stores

295. A 36 year old woman develops fever, chills and flank pain. She presents to the ER and is
diagnosed with pyelonephritis. What is the most likely causative organism?

a) E. coli

b) Chlamydia

c) S. pneumonia

d) S. aureus

296. Microalbuminuria refers to urinary albumin excretion rate of:


a. 30-300 mg/24 hour
b. 400-600 mg/24 hour
c. 700-900 mg/24 hour
d. > 100 mg/24 hour

297. A 41 year old man is seen for hematuria. He states that he has had this on two previous occasions,
both in relation to an upper respiratory tract infection. On both previous occasions the urine cleared
spontaneously over a period of five to seven days.

50
At the time of the present visit he states that he has been feeling lethargic with a sore throat for the past five days.
Blood pressure is 170/95, urine analysis shows specific gravity 1020, nitrites negative, +++ blood and +++ protein.
Urine microscopy reveals oxalate crystals, dysmorphic red blood cells and red cell casts. The presence of
dysmorphic red blood cells is indicative of:

a) Urine infection

b) Delay in analysis of the urine sample

c) Glomerular bleeding

d) Urothelial malignancy

e) Urinary tract calculus

298. General urinalysis in chronic pyelonephritis characterized by:

a) massive proteinuria;

b) leucocyturia;

c) hematuria;

d) nephrotic syndrome;

299. In which one of the following conditions, is a renal biopsy contraindicated-

a) Acute renal failure

b) Uncontrolled hypertension

c) Nephrotic syndrome

d) Isolated hematuria

300. A 65 year old woman presents with generalized

edema, skin ulceration and hypertension. Urine examination shows subnephrotic proteinuria (< 2gm) and
microscopic haematuria. Serum complement levels are decreased and she is positive for antihepatitisс antibodies.
The likely diagnosis is -

a) PSGN

b) Essential mixed cryoglobulinemia

c) Membrano proliferative glomerulonephritis

d) Focal segmental glomerulosclerosis

301. The indication for the purpose of glucocorticoids in chronic glomerulonephritis is:

a. nephrotic syndrome

b. hypertension

c. renal failure

d. hematuria

302. All of the symptoms characteristic of glomerulonephritis

a. Hypertension, the urine changes, edema

b. Pain in the lumbar region, edema, dysuria

51
c. dysuria, bleeding, pain

d. itching, dysuria, hypertension

303. Тhe treatment of chronic pyelonephritis is used:

a. antibiotics

b. antiplatelet agents

c. kortikosterovdov

d. methylxanthines

304. In glomerulonephritis affected:

a. tubules

b. glomerulus

c. renal pelvis

d. vessels

305. The most common infectious agent associated with chronic pyelonephritis is -

a) Proteus vulgaris

b) Klebsiella pneumonia

c) Staphylococcus aureus

d) Escherichia coli

306. In pyelonephritis the diagnostic urinary finidng is -

a) RBC cast

c) Pus cells d) RBCs

307. Microalbuminuria refers to urinary albumin excretion rate of -

a) 30-300mg/24 hour b) 400-600 mg/24 hour

c) 700-900 mg/24 hour d) >1000 mg/24 hour

308. A 21-year-old woman presents with hypertension, fatigue, and mi-

croscopic hematuria. A renal biopsy demonstrates glomerulonephritis second-

ary to focal segmental glomerulosclerosis. Which of the following would be

the most appropriate step to take next in the treatment of this patient's disease:
a) renal dialysis;

b) cyclosporine;

c) prednisone;

52
d) cyclophosphamide;

309. A man has a BP of 160/90mmHg, proteinuria++. KUB US are equally reduced in


size with smooth borders and normal pelvic calyceal system. What is the cause
of HTN in the pt?
a. Chronic glomerulonephritis
b. Chronic pyelonephritis
c. Bilateral renal artery stenosis
d. Essential HTN
e. Polycystic kidney

310. 30yo man complains of vague pain in the loin with BP=140/90mmHg. He is
found to have proteinuria and hematuria. What is the inv to confirm the dx?
a. USG
b. ANCA
c. ANA
d. Urine microscopy and culture
e. Stool culture

311. Which one of the following statements regarding pyelonephritis is correct?


a. Recurrent infection with the same strain usually
occurring after one week of cessation of therapy.
b. Xantho-granulomatous pyelonephritis is seen with diabetes mellitus
c. Emphysematous pyelonephritis is seen with staghorn calculus
d. 10,000 viable bacteria per ml in clean voided midstream urine is of significance

312. In hematuria of glomerular origin the urine is characterized by the presence of all of the following except:
a. Red cell casts
b. Acanthocytes
c. Crenated red cells
d. Dysmorphic red cells

313. True about Post-Streptococcal Glomerulonephritis is:


a. 50% of cases occur after pharyngitis
b. Early treatment of Pharyngitis eliminates the risk of
P.S.G.N.
c. Glomerulonephritis, secondary to skin infection, is more common in summer

314. Renal vein thrombosis is most commonly associated


with:
a. Diabetic nephropathy
b. Membranous glomerulopathy
c. Minimal change disease
d. Membrano- proliferative glomerulonephritis
d. Recurrence is seen

315. Non - selective proteinuria is seen in:


a. Minimal change
b. Mesangio - proliferative GN
c. Membranous glomerulonephritis
d. Focal segmental Glomerulosclerosis

316. Post-infective glomerulonephritis present as:


a- ARF
b. Nephrotic syndrome
c. Nephritic syndrome
d. Asymptomatic hematuria

53
317. Leukocyturia is the most characteristic symptom in:
a) pyelonephritis
b) glomerulonephritis
c) amyloidosis
d) nephrolithiasis
e) polycystic

318. Renal vein thrombosis is most commonly associated with:


A. Diabetic nephropathy
B. Membranous glomerulonephritis
C. Minimal change disease
D. Membranoproliferative glomerulonephritis

319. A 40-year-old patient with tonsillitis developed edema on the 5th day of the disease, macrohematuria, and
increased blood pressure. Most likely diagnosis:
a) acute glomerulonephritis
b) acute pyelonephritis
c) exacerbation of chronic glomerulonephritis
d) apostematic nephritis
e) renal amyloidosis

320. The most common infectious agent associated with chronic pyelonephritis is -

e) Proteus vulgaris

f) Klebsiella pneumonia

g) Staphylococcus aureus

h) Escherichia coli

321. A 35-year-old woman seeks medical attention 2 weeks after her honeymoon. During
the trip, she had burning on urination, frequency, and urgency, which she self-treated
by drinking cranberry juice. She now complains of fever and left flank pain. What is
the MOST likely diagnosis?
A. Renal colic
B. Ectopic pregnancy
C. Pyelonephritis
D. Polycystic ovarian disease
E. Renal tuberculosis

322. A 32-year-old African-American woman presents to her primary care doctor complaining of fatigue, joint
stiffness and pain, mouth ulcers, and hair loss. She has also developed joint stiffness and pain in her hands, wrists,
and knees that is present for about 1 h upon awakening. For the past 1 month, she has had an area of hair loss on her
scalp associated with a raised scaly rash. She also reports a severe “sunburn” on her face, upper neck, and back that
occurred after <1 h of sun exposure and which was unusual for her. Laboratory studies show the following:
An antinuclear antibody (ANA) is positive at a titer of 1:640. Antibodies to double-stranded DNA are negative, and
anti- Smith antibodies are positive at a titer of 1:160. The rheumatoid factor level is 37 IU/L. What is the most likely
diagnosis?
A. Behзet’s disease
B. Discoid lupus erythematosus
C. Rheumatoid arthritis
D. Sarcoidosis
E. Systemic lupus erythematosus
323. Crescentic glomerulonephritis is most associated with:
A. p-ANCA
B. HIV
C. Cytomegalovirus
D. Cryoglobulin
E. Nephritic factor

54
324. A 60-year-old man has had type II diabetes mellitus for the last 10 years. His hemoglobin
A1c has averaged 7.9% for the last 5 years. Lately, his urine dipstick has revealed proteinuria.
Pick the MOST likely description of findings on this patient’s renal biopsy.
A. Mesangial nodules and thick glomerular basement membranes
B. Crescents and vasculitis
C. Granular subepithelial immune complex deposits
D. Double contoured glomerular basement membranes “tram tracks”
E. Interstitial inflammation with eosinophils

325. A biopsy specimen that was obtained from a patient who had undergone renal transplantation
shows neutrophils in tubules in the medulla (white blood cell casts). Pick
the MOST likely diagnosis.
A. Rejection of the transplant
B. Transplant vasculopathy
C. Urinary tract infection
D. Recurrent diabetic nephropathy
E. Toxicity of immunosuppressive drugs

326. The location of the deposits in membranous glomerulonephritis suggests:


A. An antibody directed against an intrinsic antigen on podocytes
B. An antibody directed against tubular basement membranes
C. A large negatively charged preformed antibody-antigen complex
D. An antibody directed against glomerular basement membrane
E. An antibody to neutrophil granules

327. A 17-year-old patient is referred by his GP after presenting with periorbital oedema.
The patient noticed the oedematous eyes 3 days ago, but reports feeling unwell
since a throat infection 3 weeks ago with nausea and vomiting in the last week.
A urine dipstick is positive for protein and blood while serum creatinine and
urea are mildly deranged. The most likely diagnosis is:
A. Nephrotic syndrome
B. Nephritic syndrome
C. Renal failure
D. Glomerulonephritis
E. Von Grawitz tumour

328. The normal rate of glomerular filtration by endogenous clearance is:


1) 80 – 120 ml/min
2) 50 - 75 ml/min
3) 125 – 145 ml/min
4) 150 - 170 ml/min
5) 180 – 200 ml/min

329. Features of glomerular haematuria -


a) Dysmorphic R.B.C., R.B.C. cast

b) Fragmeneted R.BG.
c) Full of R.B.C. in high power field
d) Gross haematuria

330. Leukocyturia is the most characteristic symptom in:


a) pyelonephritis
b) glomerulonephritis
c) amyloidosis
d) nephrolithiasis
e) polycystic

331. Indicate the mechanism that underlies the pathogenesis of acute glomerulonephritis:
a) immunocomplex

55
b) antibody (antibodies to the glomerular basement membrane)
c) toxic damage to the kidneys
d) dystrophic changes
e) ischemic

332. Proteinuria, hypoproteinemia in combination with red blood cells and edema are most characteristic of:
a) acute glomerulonephritis
b) pyelonephritis
c) kidney stone disease
d) cystitis
e) renal amyloidosis

333. CRF with anemia best treatment -


a) Iron
b) Erythropoietin
c) Blood transfusion ;
d) Folic acid

334. Anemia that is seen in patients with chronic renal disease is usually
caused by insufficient
a. Iron stores
b. Vitamin B12
c. Renin levels
d. Erythropoietin levels
e. Folate stores

335. The most common cause of chronic renal failure is-


a. Diabetes mellitus
b. Hypertension
c. Glomerular diseases
d. Interstitial diseases

336. A 5-year-old boy presents to his GP with a 3-day history of puffy eyes. He
has been unwell with a coryzal illness for the last week. His mother states he
has had no new medications and no hayfever, allergies or asthma. On further
examination he has generalized oedema and scrotal oedema. He is tachycardiac
and has cool peripheries, no skin rashes or erythema. What is the most likely
diagnosis?
a. Periorbital cellulitis
b. Allergic reaction
c. Nephrotic syndrome
d. Nephrotic syndrome with hypovolaemia
e. C1 esterase deficiency

337. The term end stage renal disease (ESRD)' is considered appropriate when GFR falls to-
a) 50% of normal b) 25% of
normal с) 10 - 25% of normal d) 5 -10% of
normal

338. A 60yo man complains of tiredness, lethargy and itching that is severe after a hot bath. He also has nocturia,

56
polyuria and nausea and vomiting. Exam: pallor, pigmentation and generalized

57
edema. What is the single most likely dx?
a. Hyperthyroidism
b. Chronic renal failure.
c. Liver failure
d. Eczema

339. A young boy presented with bilateral periorbital edema, ankle swelling and increase in body weight. What is
the most likely dx?
a. Chronic heart failure
b. Nephrotic syndrome
c. Renal failure
d. Acute heart failure
e. Glomerulonephritis

340. Which of the following statements regarding aminoglycosides is true?


A) Liver function should be followed closely during administration.
B) Volume of distribution is increased in obese patients.
C) Nephrotoxic effects can occur with administration.
D) Lupus-like syndrome can occur with prolonged use.
E) Respiratory depression is not associated with aminoglycosides.

341. In CRF there is-


a) Oliguria
b) Polyuria
c) Olijuria during day and polyuria during night
d) Anuria

342. A 15yo boy presents with generalized edema. His urinalysis reveals protein
+++, eGFR =110. What is the most likely dx?
a. IgA nephropathy
b. Membranous nephropathy
c. Minimal change disease

d. Lupus nephritis

343. A 35yo man presented with hematuria, abdominal swelling and has a BP of
190/140. What is the most diagnostic inv?
a. Cystoscopy
b. USG
c. CT
d. Renal biopsy

344. A 32yo man develops hematuria 2wks after a sore throat. What is the
dx?
a. Post infection nephritis
b. IgA nephropathy
c. Membranous nephritis
d. Glomerulonephritis

345. Indicate the mechanism that underlies the pathogenesis of acute glomerulonephritis:
1. immunocomplex
2. antibody (antibodies to the glomerular basement membrane)
3. toxic damage to the kidneys
4. dystrophic changes
5. ischemic

346. Proteinuria, hypoproteinemia in combination with red blood cells and edema are most characteristic of:

58
1. acute glomerulonephritis
2. pyelonephritis
3. kidney stone disease
4. cystitis
5. renal amyloidosis
347. RBC cast in the microscopic examination of the urine is an indicator of:
a. Acute glomerulonephritis
b. Acute pyelonephritis
c. Chronic glomerulonephritis
d . Nephrotic syndrome

348. Manifestation of acute glomerulonephritis includes each of the following except:


a. Peri-orbital edema
b. Hypertensive encephalopathy
c. Acute renal failure
d. Optic atrophy

349. What does "uremia" mean?

a. The increase in blood urea level

b. The increase in urea and creatinine

C.The clinical manifestations of toxicity associated with renal failure

d. violation of the acid-base status

350. A 22-year-old sexually active woman presents with a 24-hour history of dysuria. She is otherwise healthy and
has no other symptoms.

Appropriate management includes

A) Urine culture followed by antibiotics until culture result is determined

B) Empiric antibiotic for 3 days

C) Continued observation

D) Midstream urinalysis followed by microscopic evaluation and

treatment if positive

351. True about Light microscopic changes in Minimal Change Glomerulonephritis is - (AI 95)
a) No abnormality
b) Fusion of foot process
c) Absence of Immunoglobulins
d) Absence of complement

352. Multiple patients present to your office with hematuria following an outbreak of Group A Streptococcus.
Biopsy reveals that all of the patients have the same disease, characterized by large, hypercellular glomeruli with
neutrophil infiltration. Which patient has the best prognosis?
a) 65-year-old nulliparous woman
b) 50-year-old man with a history of strep infection
c) 8-year-old boy who undergoes no treatment
d) 38-year-old man with sickle cell trait
e) 18-year-old man treated with corticosteroids

59
353. Post-infective glomerulonephritis present as-
a) ARF
b) Nephrotic syndrome
c) Nephritic syndrome
d) Asymptomatic hematuria

60

You might also like